Combined for Final

¡Supera tus tareas y exámenes ahora con Quizwiz!

Wyrich Corporation has two divisions: Blue Division and Gold Division. The following report is for the most recent operating period: Total CompanyBlue Division Gold DivisionSales$522,000$391,000 $131,000 Variable expenses 160,670 89,930 70,740 Contribution margin 361,330 301,070 60,260 Traceable fixed expenses 286,000 239,000 47,000 Segment margin 75,330$62,070 $13,260 Common fixed expenses 73,080 Net operating income$2,250 The Gold Division's break-even sales is closest to: $102,174$261,043$142,043$518,750

$102,174 Segment CM ratio = Segment contribution margin ÷ Segment sales = $60,260 ÷ $131,000 = 0.460 Dollar sales for a segment to break even = Traceable fixed expenses ÷ Segment CM ratio = $47,000 ÷ 0.460 = $102,174

Purves Corporation is using a predetermined overhead rate that was based on estimated total fixed manufacturing overhead of $121,000 and 10,000 direct labor-hours for the period. The company incurred actual total fixed manufacturing overhead of $113,000 and 10,900 total direct labor-hours during the period. The predetermined overhead rate is closest to: $10.37 $12.10 $11.10 $11.30

$12.10 Estimated total fixed manufacturing overhead (a)$121,000Estimated activity level (b) 10,000Predetermined overhead rate (a) ÷ (b)$12.10

Mullee Corporation produces a single product and has the following cost structure: Number of units produced each year 7,000Variable costs per unit: Direct materials$51Direct labor$12Variable manufacturing overhead$2Variable selling and administrative expense$5Fixed costs per year: Fixed manufacturing overhead$441,000Fixed selling and administrative expense$112,000 The absorption costing unit product cost is: $149 per unit$65 per unit$63 per unit$128 per unit

$128 per unit Direct materials$51 Direct labor12 Variable manufacturing overhead2 Fixed manufacturing overhead cost ($441,000 ÷ 7,000 units)=63 Absorption costing unit product cost$128

Leelanau Corporation uses a job-order costing system. The following data are for last year: Work in process beginning balance$10,500Work in process ending balance$19,000Cost of goods manufactured$323,000Direct materials$115,000Direct Labor$78,000 Leelanau applies overhead using a predetermined rate. What amount of overhead was applied to work in process last year? $138,500 $121,500 $130,000 $203,500

$138,500

Rovinsky Corporation, a company that produces and sells a single product, has provided its contribution format income statement for November. Sales (5,700 units)$319,200Variable expenses 188,100Contribution margin 131,100Fixed expenses 106,500Net operating income$24,600 If the company sells 5,300 units, its net operating income should be closest to: $24,600 $2,200 $22,874 $15,400

$15,400 Selling price per unit = Sales ÷ Quantity sold = $319,200 ÷ 5,700 units = $56 per unit Variable expenses per unit = Variable expenses ÷ Quantity sold = $188,100 ÷ 5,700 units = $33 per unit Unit CM = Selling price per unit - Variable expenses per unit = $56 per unit - $33 per unit = $23 per unit Profit = (Unit CM × Q) - Fixed expenses = ($23 per unit × 5,300 units) - $106,500 = $121,900 - $106,500 = $15,400

Inacio Corporation uses the weighted-average method in its process costing system. Data concerning the first processing department for the most recent month are listed below: Beginning work in process inventory: Units in beginning work in process inventory 800 Materials costs$12,900 Conversion costs$5,000 Percent complete with respect to materials 75%Percent complete with respect to conversion 20%Units started into production during the month 9,500 Units transferred to the next department during the month 8,400 Materials costs added during the month$172,000 Conversion costs added during the month$240,200 Ending work in process inventory: Units in ending work in process inventory 1,900 Percent complete with respect to materials 90%Percent complete with respect to conversion 30% The cost per equivalent unit for materials for the month in the first processing department is closest to: $17.01 $17.95 $16.70 $18.29

$18.29 Weighted-average method MaterialsUnits transferred to the next department 8,400 Ending work in process:Materials: 1,900 units × 90% 1,710 Equivalent units of production 10,110 MaterialsCost of beginning work in process inventory$12,900 Costs added during the period 172,000 Total cost (a)$184,900 Equivalent units of production (b) 10,110 Cost per equivalent unit (a) ÷ (b)$18.29

Bernson Corporation is using a predetermined overhead rate that was based on estimated total fixed manufacturing overhead of $492,000 and 30,000 machine-hours for the period. The company incurred actual total fixed manufacturing overhead of $517,000 and 28,300 total machine-hours during the period. The amount of manufacturing overhead that would have been applied to all jobs during the period is closest to: (Round your intermediate calculations to 2 decimal places.) $464,120 $492,000 $487,703 $25,000

$464,120 Estimated total fixed manufacturing overhead (a)$492,000Estimated activity level (b) 30,000Predetermined overhead rate (a) ÷ (b)$16.40Actual activity level 28,300Manufacturing overhead applied$464,120

Production order processing is an example of a: A) Unit-level activity. B) Batch-level activity. C) Product-level activity. D) Organization-sustaining activity.

B) Batch-level activity.

Brendal Corporation is a manufacturer that uses job-order costing. The company has supplied the following data for the just completed year: Estimated total manufacturing overhead at the beginning of the year $693,000 Estimated direct labor-hours at the beginning of the year 42,000direct labor-hours Results of operations: Raw materials (all direct) requisitioned for use in production$525,000 Direct labor cost$690,000 Actual direct labor-hours 49,000direct labor-hoursManufacturing Overhead Indirect labor cost$138,000 Other manufacturing overhead costs incurred$506,000 How much is the total manufacturing cost added to Work in Process during the year? $1,215,000 $1,803,000 $1,498,500 $2,023,500

$2,023,500 Predetermined overhead rate = Estimated total manufacturing overhead cost ÷ Estimated total amount of the allocation base = $693,000 ÷ 42,000 direct labor-hours = $16.50 per direct labor-hour Overhead applied = Predetermined overhead rate × Amount of the allocation base incurred = $16.50 per direct labor-hour × 49,000 direct labor-hours = $808,500 Direct materials$525,000Direct labor 690,000Manufacturing overhead applied 808,500Total manufacturing cost$2,023,500

Gilchrist Corporation bases its predetermined overhead rate on the estimated machine-hours for the upcoming year. At the beginning of the most recently completed year, the Corporation estimated the machine-hours for the upcoming year at 79,000 machine-hours. The estimated variable manufacturing overhead was $7.38 per machine-hour and the estimated total fixed manufacturing overhead was $2,347,090. The predetermined overhead rate for the recently completed year was closest to: $37.09 per machine-hour $36.07 per machine-hour $7.38 per machine-hour $29.71 per machine-hour

$37.09 per machine-hour Estimated total manufacturing overhead = $2,347,090 + ($7.38 per machine-hour × 79,000 machine-hours) = $2,930,110 Predetermined overhead rate = Estimated total manufacturing overhead ÷ Estimated total amount of the allocation base = $2,930,110 ÷ 79,000 machine-hours = $37.09 per machine-hour

During September at Renfro Corporation, $65,000 of raw materials were requisitioned from the storeroom for use in production. These raw materials included both direct and indirect materials. The indirect materials totaled $4,000. The journal entry to record this requisition would include a debit to Manufacturing Overhead of: $65,000 $4,000 $0 $61,000

$4,000 Work in Process61,000 +Manufacturing Overhead4,000 = Raw Materials 65,000

Pedregon Corporation has provided the following information: Cost per Unit Cost per PeriodDirect materials$6.35 Direct labor$3.75 Variable manufacturing overhead$1.50 Fixed manufacturing overhead $15,000Sales commissions$0.50 Variable administrative expense$0.55 Fixed selling and administrative expense $4,500 If 4,000 units are sold, the total variable cost is closest to: $58,400 $66,200 $50,600 $46,400

$50,600 Direct materials$6.35Direct labor 3.75Variable manufacturing overhead 1.50Sales commissions 0.50Variable administrative expense 0.55Variable cost per unit sold$12.65 Variable cost per unit sold (a)$12.65Number of units sold (b) 4,000Total variable costs (a) × (b)$50,600

Kesterson Corporation has provided the following information: Cost per UnitCost per PeriodDirect materials$6.20 Direct labor$3.10 Variable manufacturing overhead$1.35 Fixed manufacturing overhead $14,000Sales commissions$1.50 Variable administrative expense$0.40 Fixed selling and administrative expense $4,500 If 6,000 units are produced, the total amount of direct manufacturing cost incurred is closest to: $55,800 $63,900 $80,700 $64,800

$55,800 Direct materials$6.20+ Direct labor 3.10= Direct manufacturing cost per unit (a)$9.30 Number of units produced (b) 6,000 Total direct manufacturing cost (a) × (b)$55,800

The following costs were incurred in May: Direct materials$41,000 Direct labor$13,000 Manufacturing overhead$46,000 Selling expenses$18,000 Administrative expenses$15,000 Conversion costs during the month totaled: $54,000 $133,000 $59,000 $87,000

$59,000 Conversion cost = Direct labor + Manufacturing overhead = $13,000 + $46,000 = $59,000

Sorin Inc., a company that produces and sells a single product, has provided its contribution format income statement for January. Sales (4,200 units)$155,400Variable expenses 100,800Contribution margin 54,600Fixed expenses 42,400Net operating income$12,200 If the company sells 4,600 units, its total contribution margin should be closest to: $54,600 $59,800 $69,400 $13,362

$59,800 Selling price per unit = Sales ÷ Quantity sold = $155,400 ÷ 4,200 units = $37 per unit Variable expenses per unit = Variable expenses ÷ Quantity sold Variable expenses per unit = $100,800 ÷ 4,200 units = $24 per unit Unit CM = Selling price per unit - Variable expenses per unit = $37 per unit - $24 per unit = $13 per unit Total CM = Unit CM × Quantity sold = $13 per unit × 4,600 units = $59,800

Laflame Corporation uses a job-order costing system with a single plantwide predetermined overhead rate based on machine-hours. The company based its predetermined overhead rate for the current year on the following data: Total machine-hours 70,000Total fixed manufacturing overhead cost$357,000Variable manufacturing overhead per machine-hour$3.90 The estimated total manufacturing overhead is closest to: $273,000 $630,000 $357,004 $357,000

$630,000 Estimated total manufacturing overhead cost = Estimated total fixed manufacturing overhead cost + (Estimated variable overhead cost per unit of the allocation base × Estimated total amount of the allocation base) = $357,000 + ($3.90 per machine-hour × 70,000 machine-hours) = $357,000 + $273,000 = $630,000

Purchase order processing is an example of a: A) Unit-level activity. B) Batch-level activity. C) Product-level activity. D) Organization-sustaining activity.

B) Batch-level activity.

To obtain the dollar sales volume necessary to attain a given target profit, which of the following formulas should be used? (Fixed expenses + Target net profit)/Total contribution margin(Fixed expenses + Target net profit)/Contribution margin ratio Fixed expenses/Contribution margin per unitTarget net profit/Contribution margin ratio

(Fixed expenses + Target net profit)/Contribution margin ratio

Piedmont Company segments its business into two regions—North and South. The company prepared the contribution format segmented income statement as shown: Total Company North SouthSales$600,000 $400,000 $200,000Variable expenses 360,000 280,000 80,000Contribution margin 240,000 120,000 120,000Traceable fixed expenses 120,000 60,000 60,000Segment margin 120,000 $60,000 $60,000Common fixed expenses 50,000 Net operating income$70,000 Required: 1. Compute the companywide break-even point in dollar sales. 2. Compute the break-even point in dollar sales for the North region. 3. Compute the break-even point in dollar sales for the South region.

1 425,000 2 200,000 3 100,000 The companywide break-even point is computed as follows: Dollar sales for company to break even=Traceable fixed expenses + Common fixed expensesOverall CM ratio =$120,000 + $50,000$240,000 ÷ $600,000 =$170,0000.40 =$425,000 2. The break-even point for the North region is computed as follows: Dollar sales for a segment to break even=Segment traceable fixed expensesSegment CM ratio =$60,000$120,000 ÷ $400,000 =$60,0000.30 =$200,000 3. The break-even point for the South region is computed as follows: Dollar sales for a segment to break even=Segment traceable fixed expensesSegment CM ratio =$60,000$120,000 ÷ $200,000 =$60,0000.60 =$100,000

The direct labor budget of Yuvwell Corporation for the upcoming fiscal year contains the following details concerning budgeted direct labor-hours: 1st Quarter2nd Quarter3rd Quarter4th QuarterBudgeted direct labor-hours8,0008,2008,5007,800 The company uses direct labor-hours as its overhead allocation base. The variable portion of its predetermined manufacturing overhead rate is $3.25 per direct labor-hour and its total fixed manufacturing overhead is $48,000 per quarter. The only noncash item included in fixed manufacturing overhead is depreciation, which is $16,000 per quarter. Required: 1. Prepare the company's manufacturing overhead budget for the upcoming fiscal year. 2. Compute the company's predetermined overhead rate (including both variable and fixed manufacturing overhead) for the upcoming fiscal year.

1. 1stQuarter2ndQuarter3rdQuarter4thQuarterYearBudgeted direct labor-hours8,0008,2008,5007,80032,500Variable manufacturing overhead rate× $3.25× $3.25× $3.25× $3.25× $3.25Variable manufacturing overhead$26,000$26,650$27,625$25,350$105,625 2. Total budgeted manufacturing overhead for the year (a)$297,625Budgeted direct labor-hours for the year (b) 32,500Predetermined overhead rate for the year (a) ÷ (b)$9.16

The following data from the just completed year are taken from the accounting records of Mason Company: Sales$524,000Direct labor cost$70,000Raw material purchases$118,000Selling expenses$140,000Administrative expenses$63,000Manufacturing overhead applied to work in process$90,000Actual manufacturing overhead costs$80,000 InventoriesBeginningEndingRaw materials$7,000$15,000Work in process$10,000$5,000Finished goods$20,000$35,000 Required: 1. Prepare a schedule of cost of goods manufactured. Assume all raw materials used in production were direct materials. 2. Prepare a schedule of cost of goods sold. Assume that the company's underapplied or overapplied overhead is closed to Cost of Goods Sold. 3. Prepare an income statement.

1. DM Beg RM inv 7,000 Add Purchase RM 118,000 Total RM 125,000 Less end RM inv (15,000) RM used 110,000 DL 70,000 Man over WIP inv 90,000 Total man costs 270,000 Add beg WIP inv 10,000 280,000 Less end WIP inv (5,000) COGM 275,000 2. Beg finished goods inv 20,000 Add COGM 275,000 COG avail 295,000 Less end finished goods inv (35,000) Unadj COGS 260,000 Less overapplied over (10,000) Adjusted COGS 250,000 Actual manufacturing overhead cost of $80,000 ‒ Manufacturing overhead applied of $90,000 = Overapplied overhead of $10,000. 3. sales 524,000 cogs 250,000 gross margin 274,000 operating expenses selling expenses 140,000 admin expenses 63,000 203,000 NOI 71,000 Cost of goods sold ($260,000 − $10,000) = $250,000

Mauro Products distributes a single product, a woven basket whose selling price is $15 per unit and whose variable expense is $12 per unit. The company's monthly fixed expense is $4,200. Required: 1. Calculate the company's break-even point in unit sales. 2. Calculate the company's break-even point in dollar sales. 3. If the company's fixed expenses increase by $600, what would become the new break-even point in unit sales? In dollar sales?

1. The break-even point in unit sales, Q, is computed as follows: Profit=Unit CM × Q − Fixed expenses$0=($15 − $12) × Q − $4,200$0=($3) × Q − $4,200$3Q=$4,200Q=$4,200 ÷ $3Q=1,400 baskets 2. The break-even point in dollar sales is computed as follows: Unit sales to break even (a) 1,400Selling price per unit (b)$15Dollar sales to break even (a) × (b)$21,000 3. The new break-even point in unit sales, Q, is computed as follows: Profit=Unit CM × Q − Fixed expenses$0=($15 − $12) × Q − $4,800$0=($3) × Q − $4,800$3Q=$4,800Q=$4,800 ÷ $3Q=1,600 baskets The break-even point in dollar sales is computed as follows: Unit sales to break even (a) 1,600Selling price per unit (b)$15Dollar sales to break even (a) × (b)$24,000

Engberg Company installs lawn sod in home yards. The company's most recent monthly contribution format income statement follows: Amount Percent ofSalesSales$80,000 100%Variable expenses 32,000 40%Contribution margin 48,000 60%Fixed expenses 38,000 Net operating income$10,000 Required: 1. What is the company's degree of operating leverage? 2. Using the degree of operating leverage, estimate the impact on net operating income of a 5% increase in sales. 3. Construct a new contribution format income statement for the company assuming a 5% increase in sales.

1. The company's degree of operating leverage would be computed as follows: Contribution margin (a)$48,000Net operating income (b)$10,000Degree of operating leverage (a) ÷ (b) 4.8 2. A 5% increase in sales should result in a 24% increase in net operating income, computed as follows: Degree of operating leverage (a)4.8 Percent increase in sales (b)5%Estimated percent increase in net operating income (a) × (b)24% 3. The new income statement reflecting the change in sales is: Net operating income reflecting change in sales$12,400 Original net operating income (a) 10,000 Change in net operating income (b)$2,400 Percent change in net operating income (b) ÷ (a) 24%

Lin Corporation has a single product whose selling price is $120 per unit and whose variable expense is $80 per unit. The company's monthly fixed expense is $50,000. Required: 1. Calculate the unit sales needed to attain a target profit of $10,000. 2. Calculate the dollar sales needed to attain a target profit of $15,000

1. The required unit sales, Q, to attain the target profit is computed as follows: Profit=Unit CM × Q − Fixed expenses$10,000=($120 − $80) × Q − $50,000$10,000=($40) × Q − $50,000$40 × Q=$10,000 + $50,000Q=$60,000 ÷ $40Q=1,500 units 2. One approach to solving this requirement is to compute the unit sales required to attain the target profit and then multiply this quantity by the selling price per unit: Profit=Unit CM × Q − Fixed expenses$15,000=($120 − $80) × Q − $50,000$15,000=($40) × Q − $50,000$40 × Q=$15,000 + $50,000Q=$65,000 ÷ $40Q=1,625 units Unit sales to attain the target profit (a) 1,625Selling price per unit (b)$120Dollar sales to attain target profit (a) × (b)$195,000

Whirly Corporation's contribution format income statement for the most recent month is shown below: Total Per UnitSales (10,000 units)$350,000 $35.00 Variable expenses 200,000 20.00 Contribution margin 150,000 $15.00 Fixed expenses 135,000 Net operating income$15,000 Required: (Consider each case independently): 1. What would be the revised net operating income per month if the sales volume increases by 100 units? 2. What would be the revised net operating income per month if the sales volume decreases by 100 units? 3. What would be the revised net operating income per month if the sales volume is 9,000 units?

1. The revised net operating income would be: Total Per UnitSales (10,100 units)$353,500 $35.00 Variable expenses 202,000 20.00 Contribution margin 151,500 $15.00 Fixed expenses 135,000 Net operating income$16,500 2. The revised net operating income would be: Total Per UnitSales (9,900 units)$346,500 $35.00 Variable expenses 198,000 20.00 Contribution margin 148,500 $15.00 Fixed expenses 135,000 Net operating income$13,500 3. The revised net operating income would be: Total Per UnitSales (9,000 units)$315,000 $35.00 Variable expenses 180,000 20.00 Contribution margin 135,000 $15.00 Fixed expenses 135,000 Net operating income$0

Molander Corporation is a distributor of a sun umbrella used at resort hotels. Data concerning the next month's budget appear below: Selling price per unit$30Variable expense per unit$20Fixed expense per month$7,500Unit sales per month 1,000 Required: 1. What is the company's margin of safety? 2. What is the company's margin of safety as a percentage of its sales?

1. To compute the margin of safety, we must first compute the break-even unit sales. Profit=Unit CM × Q − Fixed expenses$0=($30 − $20) × Q − $7,500$0=($10) × Q − $7,500$10Q=$7,500Q=$7,500 ÷ $10Q=750 units; or, at $30 per unit, $22,500 Sales (at the budgeted volume of 1,000 units)$30,000Less break-even sales (at 750 units) 22,500Margin of safety (in dollars)$7,500 2. The margin of safety as a percentage of sales is as follows: Margin of safety (in dollars) (a)$7,500 Sales (b)$30,000 Margin of safety percentage (a) ÷ (b) 25%

Ida Sidha Karya Company is a family-owned company located in the village of Gianyar on the island of Bali in Indonesia. The company produces a handcrafted Balinese musical instrument called a gamelan that is similar to a xylophone. The gamelans are sold for $850. Selected data for the company's operations last year follow: Units in beginning inventory 0Units produced 250Units sold 225Units in ending inventory 25Variable costs per unit: Direct materials$100Direct labor$320Variable manufacturing overhead$40Variable selling and administrative$20Fixed costs: Fixed manufacturing overhead$60,000Fixed selling and administrative$20,000 The absorption costing income statement prepared by the company's accountant for last year appears below: Sales$191,250Cost of goods sold 157,500Gross margin 33,750Selling and administrative expense 24,500Net operating income$9,250 Required: 1. Under absorption costing, how much fixed manufacturing overhead cost is included in the company's inventory at the end of last year? 2. Prepare an income statement for last year using variable costing.

1. 6,000 2. income stmt sales 191,250 var expenses var man over 103,500 var sell admin 4,500 108,000 cont margin 83,250 fixed expenses fixed man over 60,000 fixed sell admin 20,000 80,000 net op income 3,250 Fixed manufacturing overhead cost deferred in inventory = 25 units in ending inventory ×$240 per unit* =$6,000 * $60,000 ÷ 250 units = $240 per unit 2. Variable cost of goods sold (225 units sold × $460* per unit) = $103,500 Variable selling and administrative expenses (225 units × $20 per unit) = $4,500 *Variable cost of goods sold per unit: Direct materials$100Direct labor 320Variable manufacturing overhead 40Variable costing unit product cost$460

The PC Works assembles custom computers from components supplied by various manufacturers. The company is very small and its assembly shop and retail sales store are housed in a single facility in a Redmond, Washington, industrial park. Listed below are some of the costs that are incurred at the company. Required: For each cost, indicate whether it would most likely be classified as direct materials, direct labor, manufacturing overhead, selling, or an administrative cost. 1.The cost of a hard drive installed in a computer. multiple choice 1Direct labor costDirect materials cost Manufacturing overhead costSelling costAdministrative cost 2. The cost of advertising in the Puget Sound Computer User newspaper. multiple choice 2Direct labor costDirect materials costManufacturing overhead costSelling cost Administrative cost 3.The wages of employees who assemble computers from components. multiple choice 3Direct labor cost Direct materials costManufacturing overhead costSelling costAdministrative cost 4.Sales commissions paid to the company's salespeople. multiple choice 4Direct labor costDirect materials costManufacturing overhead costSelling cost Administrative cost 5.The salary of the assembly shop's supervisor. multiple choice 5Direct labor costDirect materials costManufacturing overhead cost Selling costAdministrative cost 6.The salary of the company's accountant. multiple choice 6Direct labor costDirect materials costManufacturing overhead costSelling costAdministrative cost 7.Depreciation on equipment used to test assembled computers before release to customers. multiple choice 7Direct labor costDirect materials costManufacturing overhead cost Selling costAdministrative cost

1. Direct materials cost 2. Selling cost 3. Direct labor cost 4. Selling cost 5. Manufacturing overhead cost 6. Administrative cost 7. Manufacturing overhead cost

Primare Corporation has provided the following data concerning last month's manufacturing operations. Purchases of raw materials$30,000Indirect materials included in manufacturing overhead$5,000Direct labor$58,000Manufacturing overhead applied to work in process$87,000Underapplied overhead$4,000 InventoriesBeginningEndingRaw materials$12,000$18,000Work in process$56,000$65,000Finished goods$35,000$42,000 Required: 1. Prepare a schedule of cost of goods manufactured for the month. 2. Prepare a schedule of cost of goods sold for the month. Assume the underapplied or overapplied overhead is closed to Cost of Goods Sold.

1. Direct materials: Beginning raw materials inventory $12,000 Add: Purchases of raw materials 30,000 Total raw materials available 42,000 Less: Ending raw materials inventory -18,000 Raw materials used in production 24,000 Less: Indirect materials included in manufacturing overhead -5,000 $19,000 Direct labor 58,000 Manufacturing overhead applied to work in process 87,000 Total manufacturing costs 164,000 Add: Beginning work in process inventory 56,000 220,000 Less: Ending work in process inventory -65,000 Cost of goods manufactured 155,000 2. Beg finished goods inv 35,000 Add COGM 155,000 COG avail 190,000 Less end finished goods inv (42,000) Unadj COGS 148,000 Add Underapplied over 4,000 Adjusted COGS 152,000

Cherokee Inc. is a merchandiser that provided the following information: Amount Number of units sold 20,000Selling price per unit$30Variable selling expense per unit$4Variable administrative expense per unit$2Total fixed selling expense$40,000Total fixed administrative expense$30,000Beginning merchandise inventory$24,000Ending merchandise inventory$44,000Merchandise purchases$180,000 Required: 1. Prepare a traditional income statement. 2. Prepare a contribution format income statement.

1. Salesselected answer correct$600,000selected answer correctCost of goods soldselected answer correct160,000selected answer correctGross marginselected answer correct440,000selected answer correctSelling and administrative expenses:Selling expensesselected answer correct$120,000selected answer correctAdministrative expensesselected answer correct70,000selected answer correctnot attemptednot attemptednot attemptednot attempted190,000Net operating incomeselected answer correct$250,000 1. Sales: ($30 per unit × 20,000 units) = $600,000 Cost of goods sold: ($24,000 + $180,000 - $44,000) = $160,000 Selling expenses: (($4 per unit × 20,000 units) + $40,000) = $120,000 Administrative expenses: (($2 per unit × 20,000 units) + $30,000) = $70,000 2. Salesselected answer correct$600,000selected answer correctVariable expenses:Cost of goods soldselected answer correct160,000selected answer correctSelling expensesselected answer correct80,000selected answer correctAdministrative expensesselected answer correct40,000selected answer correctnot attemptednot attemptednot attemptednot attempted280,000Contribution marginselected answer correct320,000selected answer correctFixed expenses:Selling expensesselected answer correct40,000selected answer correctAdministrative expensesselected answer correct30,000selected answer correctnot attemptednot attemptednot attemptednot attempted70,000Net operating incomeselected answer correct$250,000 2. Cost of goods sold: ($24,000 + $180,000 - $44,000) = $160,000 Selling expenses: ($4 per unit × 20,000 units) = $80,000 Administrative expenses: ($2 per unit × 20,000 units) = $40,000

Last month when Holiday Creations, Inc., sold 50,000 units, total sales were $200,000, total variable expenses were $120,000, and fixed expenses were $65,000. Required: 1. What is the company's contribution margin (CM) ratio? 2. What is the estimated change in the company's net operating income if it can increase total sales by $1,000?

1. The company's contribution margin (CM) ratio is: Total sales$200,000 Total variable expenses 120,000 Total contribution margin (a)$80,000 Total contribution margin (a)$80,000 Total sales (b)$200,000 CM ratio (a) ÷ (b) 40% 2. The change in net operating income from an increase in total sales of $1,000 can be estimated by using the CM ratio as follows: Change in total sales (a)$1,000 CM ratio (b) 40%Estimated change in net operating income (a) × (b)$400

The Polaris Company uses a job-order costing system. The following transactions occurred in October: Raw materials purchased on account, $210,000. Raw materials used in production, $190,000 ($178,000 direct materials and $12,000 indirect materials). Accrued direct labor cost of $90,000 and indirect labor cost of $110,000. Depreciation recorded on factory equipment, $40,000. Other manufacturing overhead costs accrued during October, $70,000. The company applies manufacturing overhead cost to production using a predetermined rate of $8 per machine-hour. A total of 30,000 machine-hours were used in October. Jobs costing $520,000 according to their job cost sheets were completed during October and transferred to Finished Goods. Jobs that had cost $480,000 to complete according to their job cost sheets were shipped to customers during the month. These jobs were sold on account at 25% above cost. Required: 1. Prepare journal entries to record the transactions given above. 2. Prepare T-accounts for Manufacturing Overhead and Work in Process. Post the relevant transactions from above to each account. Compute the ending balance in each account, assuming that Work in Process has a beginning balance of $42,000.

1. a RM 210,000 AP 210,000 b WIP 178,000 Man Over 12,000 RM 190,000 c WIP 90,000 Man Over 110,000 S&W Pay. 200,000 d Man Over 40,000 Acc Dep 40,000 e Man Over 70,000 AP 70,000 f WIP 240,000 Man Over 240,000 g Finished G 520,000 WIP 520,000 h COGS 480,000 Finished G 480,000 h2 AR 600,000 Sales 600,000 Manufacturing overhead applied: 30,000 MH × $8 per MH = $240,000. h(2). Accounts receivable: $480,000 × 1.25 = $600,000. 2. Man Over b 12,000 240,000 f c 110,000 d 40,000 e 70,000 End Bal 8,000 WIP Beg bal 42,000 b 178,000 c 90,000 f 240,000 520,000 g End Bal 30,000

Suppose that you have been given a summer job as an intern at Issac Aircams, a company that manufactures sophisticated spy cameras for remote-controlled military reconnaissance aircraft. The company, which is privately owned, has approached a bank for a loan to help finance its growth. The bank requires financial statements before approving the loan. Required: Classify each cost listed below as either a product cost or a period cost for the purpose of preparing financial statements for the bank.

1.Depreciation on salespersons' cars.Period Costselected answer correct2.Rent on equipment used in the factory.Product Costselected answer correct3.Lubricants used for machine maintenance.Product Costselected answer correct4.Salaries of personnel who work in the finished goods warehouse.Period Costselected answer correct5.Soap and paper towels used by factory workers at the end of a shift.Product Costselected answer correct6.Factory supervisors' salaries.Product Costselected answer correct7.Heat, water, and power consumed in the factory.Product Costselected answer correct8.Materials used for boxing products for shipment overseas. (Units are not normally boxed.)Period Costselected answer correct9.Advertising costs.Period Costselected answer correct10.Workers' compensation insurance for factory employees.Product Costselected answer correct11.Depreciation on chairs and tables in the factory lunchroom.Product Costselected answer correct12.The wages of the receptionist in the administrative offices.Period Costselected answer correct13.Cost of leasing the corporate jet used by the company's executives.Period Costselected answer correct14.The cost of renting rooms at a Florida resort for the annual sales conference.Period Costselected answer correct15.The cost of packaging the company's product.Product Cost

Mickley Company's plantwide predetermined overhead rate is $14.00 per direct labor-hour and its direct labor wage rate is $17.00 per hour. The following information pertains to Job A-500: Direct materials$231Direct labor$153 Required: 1. What is the total manufacturing cost assigned to Job A-500? 2. If Job A-500 consists of 40 units, what is the unit product cost for this job? (Round your answer to 2 decimal places.)

1.Total manufacturing cost $510 2.Unit product cost $12.75 per unit 1. Total direct labor-hours required for Job A-500: Direct labor cost (a)$153Direct labor wage rate per hour (b)$17Total direct labor hours (a) ÷ (b) 9 Total manufacturing cost assigned to Job A-500: Direct materials$231Direct labor 153Manufacturing overhead applied ($14 per DLH × 9 DLHs) 126Total manufacturing cost$510 2. Unit product cost for Job A-500: Total manufacturing cost (a)$510Number of units in the job (b) 40Unit product cost (a) ÷ (b)$12.75

Fickel Company has two manufacturing departments—Assembly and Testing & Packaging. The predetermined overhead rates in Assembly and Testing & Packaging are $16.00 per direct labor-hour and $12.00 per direct labor-hour, respectively. The company's direct labor wage rate is $20.00 per hour. The following information pertains to Job N-60: Assembly Testing & PackagingDirect materials$340 $25 Direct labor$180 $40 Required: 1. What is the total manufacturing cost assigned to Job N-60? 2. If Job N-60 consists of 10 units, what is the unit product cost for this job? (Round your answer to 2 decimal places.)

1.Total manufacturing cost $753 2.Unit product cost $75.30 per unit The total direct labor-hours required for Job N-60: Assembly Testing & PackagingDirect labor cost (a)$180 $40 Direct labor wage rate per hour (b)$20 $20 Total direct labor hours (a) ÷ (b) 9 2 The total manufacturing cost and unit product cost for Job N-60 is computed as follows: Direct materials ($340 + $25) $365 Direct labor ($180 + $40) 220 Assembly Department($16 per DLH × 9 DLHs)$144 Testing & Packaging Department($12 per DLH × 2 DLHs) 24 168 Total manufacturing cost $753 Total manufacturing cost (a) $753 Number of units in the job (b) 10 Unit product cost (a) ÷ (b) $75.30

The production manager of Rordan Corporation has submitted the following quarterly production forecast for the upcoming fiscal year: 1st Quarter2nd Quarter3rd Quarter4th QuarterUnits to be produced8,0006,5007,0007,500 Each unit requires 0.35 direct labor-hours, and direct laborers are paid $12.00 per hour. Required: 1. Prepare the company's direct labor budget for the upcoming fiscal year. Assume that the direct labor workforce is adjusted each quarter to match the number of hours required to produce the forecasted number of units produced. 2. Prepare the company's direct labor budget for the upcoming fiscal year, assuming that the direct labor workforce is not adjusted each quarter. Instead, assume that the company's direct labor workforce consists of permanent employees who are guaranteed to be paid for at least 2,600 hours of work each quarter. If the number of required direct labor-hours is less than this number, the workers are paid for 2,600 hours anyway. Any hours worked in excess of 2,600 hours in a quarter are paid at the rate of 1.5 times the normal hourly rate for direct

2. Wages for regular hours: 1st Quarter = 2,600 × $12 per hour = $31,200 2nd Quarter = 2,600 × $12 per hour = $31,200 3rd Quarter = 2,600 × $12 per hour = $31,200 4th Quarter = 2,600 × $12 per hour = $31,200 Overtime wages: 1st Quarter = 200 × $12 per hour × 1.5 = $3,600 2nd Quarter = 0 × $12 per hour × 1.5 = $0 3rd Quarter = 0 × $12 per hour × 1.5 = $0 4th Quarter = 25 × $12 per hour × 1.5 = $450

The Werner Corporation uses the weighted-average method in its process costing system. The company recorded 24,400 equivalent units for conversion costs for November in a particular department. There were 4,000 units in the ending work in process inventory on November 30 which were 60% complete with respect to conversion costs. The November 1 work in process inventory consisted of 5,000 units which were 40% complete with respect to conversion costs. A total of 22,000 units were completed and transferred out of the department during the month. The number of units started during November in the department was: 22,000 units 17,000 units 26,000 units 21,000 units

21,000 units Beginning work in process inventory + Units started into production = Ending work in process inventory + Units completed and transferred out Units started into production = Ending work in process inventory + Units completed and transferred out - Beginning work in process inventory Units started into production = 4,000 + 22,000 - 5,000 = 21,000

The Assembly Department started the month with 24,000 units in its beginning work in process inventory. An additional 309,000 units were transferred in from the prior department during the month to begin processing in the Assembly Department. There were 29,000 units in the ending work in process inventory of the Assembly Department. How many units were transferred to the next processing department during the month? 333,000 362,000 304,000 314,000

304,000 Units completed and transferred out = Units in beginning work in process inventory + Units started into production or transferred in - Units in ending work in process inventory Units completed and transferred out = 24,000 + 309,000 - 29,000 = 304,000

Jersey Corporation has a process costing system in which it uses the weighted-average method. The equivalent units for conversion costs for the month were 47,500 units. The beginning work in process inventory consisted of 15,000 units, 60% complete with respect to conversion costs. The ending work in process inventory consisted of 10,000 units, 75% complete with respect to conversion costs. The number of units started during the month was: 25,000 units 34,000 units 35,000 units 40,000 units

35,000 units Weighted-average method Equivalent units of production = Units transferred to the next department or to finished goods + Equivalent units in ending work in process inventory 47,500 = Units transferred to the next department or to finished goods + 10,000 × 75% Units transferred to the next department or to finished goods = 47,500 − 10,000 × 75% = 40,000 Units in beginning work in process inventory + Units started into production or transferred in = Units in ending work in process inventory + Units completed and transferred out 15,000 + Units started into production or transferred in = 10,000 + 40,000 Units started into production or transferred in = 10,000 + 40,000 - 15,000 = 35,000

Bims Corporation uses the weighted-average method in its process costing system. The Assembly Department started the month with 2,000 units in its beginning work in process inventory that were 70% complete with respect to conversion costs. An additional 61,000 units were transferred in from the prior department during the month to begin processing in the Assembly Department. There were 18,000 units in the ending work in process inventory of the Assembly Department that were 60% complete with respect to conversion costs. What were the equivalent units for conversion costs in the Assembly Department for the month? 45,000 77,000 54,400 55,800

55,800 Weighted-average method Units transferred to the next department = Units in beginning work in process + Units started into production - Units in ending work in process = 2,000 + 61,000 - 18,000 = 45,000 ConversionUnits transferred to the next department 45,000 Ending work in process:Conversion: 18,000 units × 60% 10,800 Equivalent units of production 55,800

Jared Beverage Corporation uses a process costing system to collect costs related to the production of its celery flavored cola. The cola is first processed in a Mixing Department and is then transferred out and finished up in the Bottling Department. The finished cases of cola are then transferred to Finished Goods Inventory. The following information relates to the company's two departments for the month of January: Mixing BottlingCases of cola in work in process, January 1 10,000 5,000 Cases of cola completed/transferred out during January 65,000 ? Cases of cola in work in process, January 31 4,000 7,000 How many cases of cola were completed and transferred to Finished Goods Inventory during January? 70,000 63,000 58,000 77,000

63,000 Units completed and transferred out = Units in beginning work in process inventory + Units started into production or transferred in - Units in ending work in process inventory Units completed and transferred out = 5,000 + 65,000 - 7,000 = 63,000

Inacio Corporation uses the weighted-average method in its process costing system. Data concerning the first processing department for the most recent month are listed below: Beginning work in process inventory: Units in beginning work in process inventory 800 Materials costs$12,900 Conversion costs$5,000 Percent complete with respect to materials 75%Percent complete with respect to conversion 20%Units started into production during the month 9,500 Units transferred to the next department during the month 8,400 Materials costs added during the month$172,000 Conversion costs added during the month$240,200 Ending work in process inventory: Units in ending work in process inventory 1,900 Percent complete with respect to materials 90%Percent complete with respect to conversion 30% What are the equivalent units for conversion costs for the month in the first processing department? 8,400 8,970 570 10,300

8,970 Weighted-average method ConversionUnits transferred to the next department 8,400 Ending work in process:Conversion: 1,900 units × 30% 570 Equivalent units of production 8,970

Which of the following statements is correct with regard to a CVP graph? Multiple ChoiceA CVP graph shows the maximum possible profit.A CVP graph shows the break-even point as the intersection of the total sales revenue line and the total expense line. A CVP graph assumes that total expense varies in direct proportion to unit sales.A CVP graph shows the operating leverage as the gap between total sales revenue and total expense at the actual level of sales.

A CVP graph shows the break-even point as the intersection of the total sales revenue line and the total expense line.

Which of the following types of companies would typically use process costing rather than job-order costing? A small appliance repair shop.A manufacturer of commercial passenger aircraft.A specialty equipment manufacturer.A breakfast cereal manufacturer.

A breakfast cereal manufacturer.

Which of the following would be an acceptable measure of activity for a material handling activity cost pool? A) Number of material moves Weight of material moved Yes Yes B) Number of material moves Weight of material moved No Yes C) Number of material moves Weight of material moved Yes No D) Number of material moves Weight of material moved No No

A) Number of material moves Weight of material moved Yes Yes

Otool Inc. is considering using stocks of an old raw material in a special project. The special project would require all 240 kilograms of the raw material that are in stock and that originally cost the company $2,112 in total. If the company were to buy new supplies of this raw material on the open market, it would cost $9.25 per kilogram. However, the company has no other use for this raw material and would sell it at the discounted price of $8.35 per kilogram if it were not used in the special project. The sale of the raw material would involve delivery to the purchaser at a total cost of $71 for all 240 kilograms. What is the relevant cost of the 240 kilograms of the raw material when deciding whether to proceed with the special project? A) $1,933 B) $2,004 C) $2,220 D) $2,112

A) $1,933 Explanation: The company's relevant cost in this case is how much it could earn by selling the 240 kilograms at the discounted price of $8.35 per kilogram, less delivery costs of $71 for all 240 kilograms. Proceeds of sale at the discounted price: 240 kgs × $8.35 per kg $2,004 Less delivery cost 71 Relevant cost $1,993

All other things equal, which of the following would increase a division's residual income? A) Increase in expenses. B) Decrease in average operating assets. C) Increase in minimum required return. D) Decrease in net operating income.

B) Decrease in average operating assets.

Munafo Corporation is a specialty component manufacturer with idle capacity. Management would like to use its extra capacity to generate additional profits. A potential customer has offered to buy 6,500 units of component VGI. Each unit of VGI requires 1 unit of material I57 and 5 units of material M97. Data concerning these two materials follow: Material Units in Stock Original Cost Per Unit Current Market Price Per Unit Disposal Value Per Unit I57 2,400 $9.10 $9.40 $8.95 M97 33,960 $4.70 $4.70 $3.50 Material I57 is in use in many of the company's products and is routinely replenished. Material M97 is no longer used by the company in any of its normal products and existing stocks would not be replenished once they are used up. What would be the relevant cost of the materials, in total, for purposes of determining a minimum acceptable price for the order for product VGI? A) $174,850 B) $213,130 C) $213,850 D) $171,925

A) $174,850 Explanation: Materials requirements for I57: 6,500 units of VGI × 1 units of I57 per unit of VGI = 6,500 units of I57 Materials requirements for M97: 6,500 units of VGI × 5 units of M97 per unit of VGI = 32,500 units of M97 Relevant cost of I57: 6,500 units × current market price of $9.40 per unit $61,100 Relevant cost of M97: 32,500 units × disposal value of $3.50 per unit 113,750 Relevant cost of materials $174,850

Break-even analysis assumes that: Total revenue is constant.Unit variable expense is constant. Unit fixed expense is constant.Selling prices must fall in order to generate more revenue.

Unit variable expense is constant.

Mongelli Family Inn is a bed and breakfast establishment in a converted 100-year-old mansion. The Inn's guests appreciate its gourmet breakfasts and individually decorated rooms. The Inn's overhead budget for the most recent month appears below: Activity level 90 guests Variable overhead costs: Supplies $234 Laundry 315 Fixed overhead costs: Utilities 220 Salaries and wages 4,290 Depreciation 2,680 Total overhead cost $7,739 The Inn's variable overhead costs are driven by the number of guests. What would be the total budgeted overhead cost for a month if the activity level is 99 guests? A) $7,793.90 B) $61,541.00 C) $8,512.90 D) $7,739.00

A) $7,793.90 Explanation: Variable cost per guest for supplies = $234 ÷ 90 guests = $2.60 per guest Variable cost per guest for laundry = $315 ÷ 90 guests = $3.50 per guest Guests (q) 99 Supplies ($2.60q) $257.40 Laundry ($3.50q) 346.50 Utilities ($220) 220.00 Salaries and wages ($4,290) 4,290.00 Depreciation 2,680.00 Total overhead cost $7,793.90

In a sell or process further decision, consider the following costs: I. A variable production cost incurred prior to split-off. II. A variable production cost incurred after split-off. III. An avoidable fixed production cost incurred after split-off. Which of the above costs is (are) not relevant in a decision regarding whether the product should be processed further? A) Only I B) Only III C) Only I and II D) Only I and III

A) Only I

Jarvey Corporation is studying a project that would have a ten-year life and would require a $450,000 investment in equipment which has no salvage value. The project would provide net operating income each year as follows for the life of the project (Ignore income taxes.): Sales $500,000 Less cash variable expenses 200,000 Contribution margin 300,000 Less fixed expenses: Fixed cash expenses $150,000 Depreciation expenses 45,000 195,000 Net operating income $105,000 The company's required rate of return is 12%. The payback period for this project is closest to: A) 3 years B) 2 years C) 4.28 years D) 9 years

A) 3 years Explanation: Net operating income $105,000 Add: Noncash deduction for depreciation 45,000 Annual net cash inflow $150,000 Payback period = Investment required ÷ Annual net cash inflow = $450,000 ÷ $150,000 per year = 3 years

Which of the following would be considered a cash inflow in the financing activities section of the statement of cash flows? A) Issuing bonds payable. B) Receiving cash from customers. C) Sale of equipment. D) Collection of a loan made to another company.

A) Issuing bonds payable.

Which of the following statements is NOT correct concerning the Cash Budget? A) It is not necessary to prepare any other budgets before preparing the Cash Budget. B) The Cash Budget should be prepared before the Budgeted Income Statement. C) The Cash Budget should be prepared before the Budgeted Balance Sheet. D) The Cash Budget builds on earlier budgets and schedules as well as additional data.

A) It is not necessary to prepare any other budgets before preparing the Cash Budget.

Which of the following items would not be classified as an operating activity on the statement of cash flows? A) Cash received from customers. B) Dividends paid to the company's own stockholders. C) Payments to government agencies for taxes. D) Cash paid to compensate employees.

B) Dividends paid to the company's own stockholders.

In a flexible budget, what will happen to fixed costs as the activity level increases? A) The fixed cost per unit will decrease. B) The fixed cost per unit will remain unchanged. C) The fixed cost per unit will increase. D) Fixed costs are not included in a flexible budget.

A) The fixed cost per unit will decrease.

Machining a part for a product is an example of a: A) Unit-level activity. B) Batch-level activity. C) Product-level activity. D) Organization-sustaining activity.

A) Unit-level activity.

Providing the power required to run production equipment is an example of a: A) Unit-level activity. B) Batch-level activity. C) Product-level activity. D) Organization-sustaining activity.

A) Unit-level activity.

An unfavorable materials quantity variance indicates that: A) actual usage of material exceeds the standard material allowed for output. B) standard material allowed for output exceeds the actual usage of material. C) actual material price exceeds standard price. D) standard material price exceeds actual price.

A) actual usage of material exceeds the standard material allowed for output.

If the net present value of a project is zero based on a discount rate of 16%, then the internal rate of return is: A) equal to 16%. B) less than 16%. C) greater than 16%. D) cannot be determined from this data.

A) equal to 16%.

Variable manufacturing overhead is applied to products on the basis of standard direct labor-hours. If the labor efficiency variance is favorable, the variable overhead efficiency variance will be: A) favorable. B) unfavorable. C) zero. D) either favorable or unfavorable.

A) favorable.

The internal rate of return method assumes that a project's cash flows are reinvested at the: A) internal rate of return. B) simple rate of return. C) required rate of return. D) payback rate of return.

A) internal rate of return.

In a statement of cash flows, a change in an income taxes payable account would be recorded in the: A) operating activities section. B) financing activities section. C) investing activities section. D) stockholders' equity section.

A) operating activities section.

When preparing a direct materials budget, the required purchases of raw materials in units equals: A) raw materials needed to meet the production schedule + desired ending inventory of raw materials − beginning inventory of raw materials. B) raw materials needed to meet the production schedule − desired ending inventory of raw materials − beginning inventory of raw materials. C) raw materials needed to meet the production schedule − desired ending inventory of raw materials + beginning inventory of raw materials. D) raw materials needed to meet the production schedule + desired ending inventory of raw materials + beginning inventory of raw materials.

A) raw materials needed to meet the production schedule + desired ending inventory of raw materials − beginning inventory of raw materials.

Some investment opportunities that should be accepted from the viewpoint of the entire company may be rejected by a manager who is evaluated on the basis of: A) return on investment. B) residual income. C) contribution margin. D) segment margin.

A) return on investment.

An activity-based costing system that is designed for internal decision-making will not conform to generally accepted accounting principles because: A) under activity-based costing some manufacturing costs (i.e., the costs of idle capacity and organization-sustaining costs) will not be assigned to products. B) under activity-based costing the sum of all product costs does not equal the total costs of the company. C) activity-based costing has not been approved by the United Nation's International Accounting Board. D) activity-based costing results in less accurate costs than more traditional costing methods based on direct labor-hours or machine-hours.

A) under activity-based costing some manufacturing costs (i.e., the costs of idle capacity and organization-sustaining costs) will not be assigned to products.

Tomlin Corporation prepares its statement of cash flows using the indirect method. Which of the following would be subtracted from net income in the operating activities section of the statement? Increase in Accounts Receivable Decrease in Accrued Liabilities A)YesYes B)YesNo C)NoYes D)NoNo

A)YesYes

Which of the following performance measures will increase if inventory decreases and all else remains the same? Return on Investment Residual Income A)YesYes B)NoYes C)YesNo D)NoNo

A)YesYes

Which of the following will increase a company's manufacturing cycle efficiency (MCE)? Decrease in Inspection Time Decrease in Queue Time A)YesYes B)YesNo C)NoYes D)NoNo

A)YesYes

Accepting a special order will improve overall net operating income if the revenue from the special order exceeds: A) the contribution margin on the order. B) the incremental costs associated with the order. C) the variable costs associated with the order. D) the sunk costs associated with the order.

B) the incremental costs associated with the order.

Herlocker Corporation is a shipping container refurbishment company that measures its output by the number of containers refurbished. The company has provided the following fixed and variable cost estimates that it uses for budgeting purposes. Fixed Element per Month Variable Element per Container Refurbished Revenue $4,600 Employee salaries and wages $42,700 $1,100 Refurbishing materials $600 Other expenses $29,100 When the company prepared its planning budget at the beginning of February, it assumed that 26 containers would have been refurbished. The amount shown for revenue in the planning budget for February would have been closest to: A) $136,300 B) $119,600 C) $133,400 D) $122,200

B) $119,600 Explanation: Planning Budget Containers refurbished (q) 26 Revenue ($4,600q) $119,600

Buckson Framing's cost formula for its supplies cost is $1,350 per month plus $18 per frame. For the month of June, the company planned for activity of 716 frames, but the actual level of activity was 713 frames. The actual supplies cost for the month was $14,820. The supplies cost in the flexible budget for June would be closest to: A) $14,820 B) $14,184 C) $14,178 D) $14,238

B) $14,184

The following materials standards have been established for a particular product: Standard quantity per unit of output 4.6grams Standard price $15.05per gram The following data pertain to operations concerning the product for the last month: Actual materials purchased 3,100grams Actual cost of materials purchased $44,020 Actual materials used in production 2,400grams Actual output 300units What is the materials quantity variance for the month? A) $9,940 U B) $15,351 U C) $14,484 U D) $10,535 U

B) $15,351 U Explanation: SQ = 4.6 grams per unit × 300 units = 1,380 grams Materials quantity variance = (AQ - SQ) × SP = (2,400 grams - 1,380 grams) × $15.05 per gram = (1,020 grams) × $15.05 per gram = $15,351 U

Seventy percent of Pitkin Corporation's sales are collected in the month of sale, 20% in the month following sale, and 10% in the second month following sale. The following are budgeted sales data for the company: January February March April Budgeted sales $200,000 $300,000 $350,000 $250,000 Total budgeted cash collections in April would be: A) $175,000 B) $275,000 C) $70,000 D) $30,000

B) $275,000 February sales ($300,000 × 10%) $30,000 March sales ($350,000 × 20%) $70,000 April sales ($250,000 × 70%) $175,000 Total cash collections $275,000

Bugos Corporation is a service company that measures its output by the number of customers served. The company has provided the following fixed and variable cost estimates that it uses for budgeting purposes. Fixed Element per Month Variable Element per Customer Served Revenue $4,200 Employee salaries and wages $58,800 $900 Travel expenses $700 Other expenses $33,300 When the company prepared its planning budget at the beginning of March, it assumed that 40 customers would have been served. The amount shown for "Employee salaries and wages" in the planning budget for March would have been closest to: A) $91,200 B) $94,800 C) $92,600 D) $102,889

B) $94,800 Explanation: Planning Budget Customers served (q) 40 Employee salaries and wages ($58,800 + $900q) $94,800

Product X-547 is one of the joint products in a joint manufacturing process. Management is considering whether to sell X-547 at the split-off point or to process X-547 further into Xylene. The following data have been gathered: I. Selling price of X-547 II. Variable cost of processing X-547 into Xylene. III. The avoidable fixed costs of processing X-547 into Xylene. IV. The selling price of Xylene. V. The joint cost of the process from which X-547 is produced. Which of the above items are relevant in a decision of whether to sell the X-547 as is or process it further into Xylene? A) I, II, and IV. B) I, II, III, and IV. C) II, III, and V. D) I, II, III, and V

B) I, II, III, and IV.

Which one of the following transactions should be classified as a financing activity on the statement of cash flows? A) Purchase of equipment. B) Purchase of the company's own stock. C) Sale of a long-term investment. D) Payment of interest to a lender.

B) Purchase of the company's own stock.

The general model for calculating a quantity variance is: A) Actual quantity of inputs used × (Actual price − Standard price). B) Standard price × (Actual quantity of inputs used − Standard quantity allowed for output). C) (Actual quantity of inputs used × Actual price) − (Standard quantity allowed for output × Standard price). D) Actual price × (Actual quantity of inputs used − Standard quantity allowed for output).

B) Standard price × (Actual quantity of inputs used − Standard quantity allowed for output).

Which of the following statements is NOT correct concerning the Manufacturing Overhead Budget? A) The Manufacturing Overhead Budget provides a schedule of all costs of production other than direct materials and labor costs. B) The Manufacturing Overhead Budget shows only the variable portion of manufacturing overhead. C) The Manufacturing Overhead Budget shows the expected cash disbursements for manufacturing overhead. D) The Manufacturing Overhead Budget is prepared after the Sales Budget.

B) The Manufacturing Overhead Budget shows only the variable portion of manufacturing overhead.

In a statement of cash flows, which of the following would be classified as an investing activity? A) The sale of the company's own common stock for cash. B) The sale of equipment. C) Interest paid to a lender. D) The issuance of bonds payable.

B) The sale of equipment.

A company has unlimited funds to invest at its discount rate. The company should invest in all projects having: A) an internal rate of return greater than zero. B) a net present value greater than zero. C) a simple rate of return greater than the discount rate. D) a payback period less than the project's estimated life.

B) a net present value greater than zero.

In a statement of cash flows, issuing bonds payable affects the: A) operating activities section. B) financing activities section. C) investing activities section. D) free cash flow activities section.

B) financing activities section.

A budget that is based on the actual activity of a period is known as a: A) continuous budget. B) flexible budget. C) static budget. D) master budget.

B) flexible budget.

The production department should generally be responsible for materials price variances that resulted from: A) purchases made in uneconomical lot-sizes. B) rush orders arising from poor scheduling. C) purchase of the wrong grade of materials. D) changes in the market prices of raw materials.

B) rush orders arising from poor scheduling.

Which of the following costs are always irrelevant in decision making? A) avoidable costs B) sunk costs C) opportunity costs D) fixed costs

B) sunk costs

If variable manufacturing overhead is applied on the basis of direct labor-hours and the variable overhead rate variance is favorable, then: A) the actual variable overhead rate exceeded the standard rate. B) the standard variable overhead rate exceeded the actual rate. C) the actual direct labor-hours exceeded the standard direct labor-hours allowed for the actual output. D) the standard direct labor-hours allowed for the actual output exceeded the actual hours.

B) the standard variable overhead rate exceeded the actual rate.

The project profitability index and the internal rate of return: A) will always result in the same preference ranking for investment projects. B) will sometimes result in different preference rankings for investment projects. C) are less dependable than the payback method in ranking investment projects. D) are less dependable than net present value in ranking investment projects.

B) will sometimes result in different preference rankings for investment projects.

The Jabba Corporation manufactures the "Snack Buster" which consists of a wooden snack chip bowl with an attached porcelain dip bowl. Which of the following would be relevant in Jabba's decision to make the dip bowls or buy them from an outside supplier? Fixed overhead cost that can be eliminated if the bowls are purchased from the outside supplier The variable selling cost of the Snack Buster A)YesYes B)YesNo C)NoYes D)NoNo

B)YesNo

Factory security and assembly activities at an appliance manufacturing plant would be best classified as unit-level, batch-level, product-level, or organization-sustaining activities? A) Security Assembly Product Unit B) Security Assembly Batch Batch C) Security Assembly Organization Unit D) Security Assembly Organization Product

C) Security Assembly Organization Unit

Jeanclaude Corporation produces and sells one product. The budgeted selling price per unit is $105. Budgeted unit sales for July, August, September, and October are 7,400, 7,500, 13,800, and 15,300 units, respectively. All sales are on credit. Regarding credit sales, 40% are collected in the month of the sale and 60% in the following month. The budgeted accounts receivable balance at the end of August is closest to: A) $525,000 B) $315,000 C) $472,500 D) $787,500

C) $472,500 Explanation: The budgeted accounts receivable balance at the end of August is: August sales 7,500 units × $105 per unit (a) $787,500 Percent uncollected (b) 60% Accounts receivable (a) × (b) $472,500

Hamby Corporation is preparing a bid for a special order that would require 780 liters of material W34C. The company already has 640 liters of this raw material in stock that originally cost $8.30 per liter. Material W34C is used in the company's main product and is replenished on a periodic basis. The resale value of the existing stock of the material is $7.60 per liter. New stocks of the material can be readily purchased for $8.35 per liter. What is the relevant cost of the 780 liters of the raw material when deciding how much to bid on the special order? A) $6,481 B) $6,376 C) $6,513 D) $5,928

C) $6,513 Explanation: The relevant cost is the current market cost which is 780 liters × current market $8.35 per liter = $6,513.

Which of the following would be an argument for using the gross cost of plant and equipment as part of operating assets in return on investment computations? A) It is consistent with the computation of net operating income, which includes depreciation as an operating expense. B) It is consistent with the balance sheet presentation of plant and equipment. C) It eliminates the age of equipment as a factor in ROI computations. D) It discourages the replacement of old, worn-out equipment because of the dramatic, adverse effect on ROI.

C) It eliminates the age of equipment as a factor in ROI computations.

Suver Corporation has a standard costing system. The following data are available for June: Actual quantity of direct materials purchased 24,000pounds Standard price of direct materials $6.00per pound Material price variance $6,000Unfavorable Material quantity variance $2,400Favorable The actual price per pound of direct materials purchased in June was: A) $6.10 per pound B) $5.90 per pound C) $6.25 per pound D) $6.30 per pound

C) $6.25 per pound Explanation: Materials price variance = AQ × (AP - SP) $6,000 U = 24,000 pounds × (AP - $6.00 per pound) $6,000 = 24,000 pounds × (AP - $6.00 per pound) AP - $6.00 per pound = $6,000 ÷ 24,000 pounds AP - $6.00 per pound = $0.25 per pound AP = $6.00 per pound + $0.25 per pound AP = $6.25 per pound

Hodge Inc. has some material that originally cost $74,600. The material has a scrap value of $57,400 as is, but if reworked at a cost of $1,500, it could be sold for $54,400. What would be the financial advantage (disadvantage) of reworking and selling the material rather than selling it as is as scrap? A) ($79,100) B) ($21,700) C) ($4,500) D) $52,900

C) ($4,500) Explanation: Sales value of reworked material $54,400 Less: Cost to rework material 1,500 Net sales value 52,900 Current scrap value 57,400 Financial advantage (disadvantage) of reworking rather than selling as scrap $(4,500)

Piper Corporation's standards call for 1,000 direct labor-hours to produce 250 units of product. During October the company worked 1,250 direct labor-hours and produced 300 units. The standard hours allowed for October would be: A) 1,250 hours B) 1,000 hours C) 1,200 hours D) 1,300 hours

C) 1,200 hours Explanation: Standard hours per unit of output = 1,000 direct labor-hours ÷ 250 units = 4 direct labor-hours per unit Standard hours allowed = 300 units × 4 direct labor-hours per unit = 1,200 hours

Nasser Inc. reported the following results from last year's operations: Sales $12,600,000 Variable expenses 7,760,000 Contribution margin 4,840,000 Fixed expenses 3,706,000 Net operating income $1,134,000 Average operating assets $6,000,000 Last year's return on investment (ROI) was closest to: A) 9.0% B) 47.6% C) 18.9% D) 80.7%

C) 18.9% Explanation: ROI = Net operating income ÷ Average operating assets = $1,134,000 ÷ $6,000,000 = 18.9%

Othman Inc. has a $800,000 investment opportunity with the following characteristics: Sales $2,240,000 Contribution margin ratio 50% of sales Fixed expenses $1,008,000 The margin for this investment opportunity is closest to: A) 50.0% B) 45.0% C) 5.0% D) 55.0%

C) 5.0% Contribution margin (50% × $2,240,000) $1,120,000 Fixed expenses 1,008,000 Net operating income $112,000 Margin = Net operating income ÷ Sales = $112,000 ÷ $2,240,000 = 5.0%

Which of the following would be considered an operating asset in return on investment computations? A) Land being held for plant expansion. B) Treasury stock. C) Accounts receivable. D) Common stock.

C) Accounts receivable.

Which of the following will not result in an increase in return on investment (ROI), assuming other factors remain the same? A) A reduction in expenses. B) An increase in net operating income. C) An increase in operating assets. D) An increase in sales.

C) An increase in operating assets.

Guerra Electronics manufactures a variety of electronic gadgets for use in the home. Which of the following would probably be the most accurate measure of activity to use for allocating the costs of inspecting the finished products at Guerra? A) Machine-hours B) Direct labor-hours C) Inspection time D) Number of inspections

C) Inspection time

There are various budgets within the master budget. One of these budgets is the production budget. Which of the following BEST describes the production budget? A) It details the required direct labor hours. B) It details the required raw materials purchases. C) It is calculated based on the sales budget and the desired ending inventory. D) It summarizes the costs of producing units for the budget period.

C) It is calculated based on the sales budget and the desired ending inventory.

Designing a new backpack at an outdoor sports equipment company is an example of a: A) Unit-level activity. B) Batch-level activity. C) Product-level activity. D) Facility-level activity.

C) Product-level activity.

Parts administration is an example of a: A) Unit-level activity. B) Batch-level activity. C) Product-level activity. D) Organization-sustaining.

C) Product-level activity.

Testing a prototype of a new product is an example of a: A) Unit-level activity. B) Batch-level activity. C) Product-level activity. D) Organization-sustaining activity.

C) Product-level activity.

A segment of a business responsible for both revenues and expenses would be called: A) a cost center. B) an investment center. C) a profit center. D) residual income.

C) a profit center.

In a statement of cash flows, the sale of a long-term investment would ordinarily be classified as: A) an operating activity. B) a financing activity. C) an investing activity. D) a lending activity.

C) an investing activity.

Costs that can be eliminated in whole or in part if a particular business segment is discontinued are called: A) sunk costs. B) opportunity costs. C) avoidable costs. D) irrelevant costs.

C) avoidable costs.

Some investment projects require that a company increase its working capital. Under the net present value method, the investment and eventual recovery of working capital should be treated as: A) an initial cash outflow. B) a future cash inflow. C) both an initial cash outflow and a future cash inflow. D) irrelevant to the net present value analysis.

C) both an initial cash outflow and a future cash inflow.

When using a flexible budget, a decrease in activity within the relevant range: A) decreases variable cost per unit. B) increases variable cost per unit. C) decreases total costs. D) increases total costs.

C) decreases total costs.

A preference decision in capital budgeting: A) is concerned with whether a project clears the minimum required rate of return hurdle. B) comes before the screening decision. C) is concerned with determining which of several acceptable alternatives is best. D) involves using market research to determine customers' preferences.

C) is concerned with determining which of several acceptable alternatives is best.

The assumption that the cash flows from an investment project are reinvested at the company's discount rate applies to: A) both the internal rate of return and the net present value methods. B) only the internal rate of return method. C) only the net present value method. D) neither the internal rate of return nor net present value methods.

C) only the net present value method.

Departmental overhead rates may not correctly assign overhead costs due to: A) the use of direct labor hours in allocating overhead costs to products rather than machine time or quantity of materials used. B) the high correlation between direct labor-hours and the incurrence of overhead costs. C) overreliance on volume as a basis for allocating overhead costs where products differ regarding the number of units produced, lot size, or complexity of production. D) difficulties associated with identifying cost pools for the first stage of the allocation process.

C) overreliance on volume as a basis for allocating overhead costs where products differ regarding the number of units produced, lot size, or complexity of production.

The usual starting point for a master budget is: A) the direct materials purchase budget. B) the budgeted income statement. C) the sales forecast or sales budget. D) the production budget.

C) the sales forecast or sales budget.

In activity-based costing, the activity rate for an activity cost pool is computed by dividing the total overhead cost in the activity cost pool by: A) the direct labor-hours required by the product. B) the machine-hours required by the product. C) the total activity for the activity cost pool. D) the total direct labor-hours for the activity cost pool.

C) the total activity for the activity cost pool.

Rennin Dairy Corporation is considering a plant expansion decision that has an estimated useful life of 20 years. This project has an internal rate of return of 15% and a payback period of 9.6 years. How would a decrease in the expected salvage value from this project in 20 years affect the following for this project? Internal Rate of Return Payback Period A)DecreaseDecrease B)No effectDecrease C)DecreaseNo effect D)IncreaseNo effect E)No effectNo effect

C)DecreaseNo effect

Poorly trained workers could have an unfavorable effect on which of the following variances? Labor Rate Variance Materials Quantity Variance A)YesYes B)YesNo C)NoYes D)NoNo

C)NoYes

Green Thumb Gardening is a small gardening service that uses activity-based costing to estimate costs for pricing and other purposes. The proprietor of the company believes that costs are driven primarily by the size of customer lawns, the size of customer garden beds, the distance to travel to customers, and the number of customers. In addition, the costs of maintaining garden beds depends on whether the beds are low maintenance beds (mainly ordinary trees and shrubs) or high maintenance beds (mainly flowers and exotic plants). Accordingly, the company uses the five activity cost pools listed below: Activity Cost PoolActivity MeasureCaring for lawnSquare feet of lawnCaring for garden beds-low maintenanceSquare feet of low maintenance bedsCaring for garden beds-high maintenanceSquare feet of high maintenance bedsTravel to jobsMilesCustomer billing and serviceNumber of customers The company already has completed its first stage allocations of costs and has summarized its annual costs and activity as follows: Activity Cost PoolEstimatedOverheadCostExpected ActivityCaring for lawn$72,000150,000square feet of lawnCaring for garden beds-low maintenance$26,40020,000square feet of low maintenance bedsCaring for garden beds-high maintenance$41,40015,000square feet of high maintenance bedsTravel to jobs$3,25012,500milesCustomer billing and service$8,75025customers Required: Compute the activity rate for each of the activity cost pools. (Round your answers to 2 decimal places except customer billing and service.)

Caring for lawn$72,000150,000square ft. of lawn$0.48per square ft. of lawnCaring for garden beds-low maintenance$26,40020,000square ft. of low maintenance beds$1.32per square ft. of low maintenance bedsCaring for garden beds-high maintenance$41,40015,000square ft. of high maintenance beds$2.76per square ft. of high maintenance bedsTravel to jobs$3,25012,500miles$0.26per mileCustomer billing and service$8,75025customers$350per customer

Mossfeet Shoe Corporation is a single product firm. The company is predicting that a price increase next year will not cause unit sales to decrease. What effect would this price increase have on the following items for next year? ContributionMargin RatioBreak-evenPoint A)IncreaseDecrease B)DecreaseDecrease C)IncreaseNo effect D)DecreaseNo effect

Choice A

How would the following costs be classified (product or period) under variable costing at a retail clothing store? Cost of purchasing clothingSales commissionsA)ProductProductB)ProductPeriodC)PeriodProductD)PeriodPeriod

Choice B

Which of the following approaches to preparing an income statement includes a calculation of the gross margin? TraditionalApproachContributionApproachA)YesYesB)YesNoC)NoYesD)NoNo

Choice B

Which of the following would usually be found on a job cost sheet under a normal cost system? Actual direct material costActual manufacturing overhead costA)YesYesB)YesNoC)NoYesD)NoNo

Choice B

Which of the following will usually be found on an income statement prepared using absorption costing? Contribution MarginGross MarginA)YesYesB)YesNoC)NoYesD)NoNo

Choice C

Net operating income computed under variable costing would exceed net operating income computed using absorption costing if: units sold exceed units produced.units sold are less than units produced.units sold equal units produced.the average fixed cost per unit is zero.

units sold exceed units produced.

In its first year of operations, Bronfren Corporation produced 800,000 sets and sold 780,000 sets of artificial tan lines. What would have happened to net operating income in this first year under the following costing methods if Bronfren had produced 20,000 fewer sets? (Assume that Bronfren has both variable and fixed production costs.) Variable costingAbsorption costingA)No effectIncreaseB)DecreaseIncreaseC)DecreaseDecreaseD)No effectDecrease

Choice D

Wages paid to the supervisor of the warehouse where raw materials and parts are temporarily stored before being used in production is considered an example of: Direct LaborPeriod CostA)YesYesB)YesNoC)NoYesD)NoNo

Choice D

In the cost reconciliation report under the weighted-average method, the "Costs to be accounted for" section contains which of the following items? Cost of beginning work in process inventoryCost of ending work in process inventoryCost of units transferred outCost of ending finished goods inventory

Cost of beginning work in process inventory

In the cost reconciliation report under the weighted-average method, the "Total cost accounted for" equals: Cost of beginning work in process inventory + Cost of units transferred outCost of beginning work in process inventory + Cost of units transferred inCost of ending work in process inventory + Cost of units transferred outCost of ending work in process inventory + Cost added to production during the period

Cost of ending work in process inventory + Cost of units transferred out

In the Schedule of Cost of Goods Manufactured and Cost of Goods Sold, the cost of goods manufactured is computed according to which of the following equations? Cost of goods manufactured = Total manufacturing costs + Ending work in process inventory - Beginning work in process inventoryCost of goods manufactured = Total manufacturing costs + Beginning work in process inventory - Ending work in process inventory.Cost of goods manufactured = Total manufacturing costs + Beginning finished goods inventory - Ending finished goods inventoryCost of goods manufactured = Total manufacturing costs + Ending finished goods inventory - Beginning finished goods inventory

Cost of goods manufactured = Total manufacturing costs + Beginning work in process inventory - Ending work in process inventory

Corvi Corporation produces and sells one product. The budgeted selling price per unit is $126. Budgeted unit sales are shown below: July August September October Budgeted unit sales 7,300 11,500 14,200 12,100 All sales are on credit with 40% collected in the month of the sale and 60% in the following month. The expected cash collections for August is closest to: A) $551,880 B) $579,600 C) $919,800 D) $1,131,480

D) $1,131,480 Explanation: The expected cash collections for August are computed as follows: July sales: 7,300 units × $126 per unit = $919,800; $919,800 × 60% = $551,880 August sales: 11,500 units × $126 per unit = $1,449,000; $1,449,000 × 40% = 579,600 Total cash collections $1,131,480

Cosden Corporation is an oil well service company that measures its output by the number of wells serviced. The company has provided the following fixed and variable cost estimates that it uses for budgeting purposes. Fixed Element per Month Variable Element per Well Serviced Revenue $4,700 Employee salaries and wages $41,300 $1,000 Servicing materials $600 Other expenses $40,200 When the company prepared its planning budget at the beginning of May, it assumed that 29 wells would have been serviced. However, 31 wells were actually serviced during May. The total expenses in the flexible budget for May would have been closest to: A) $133,100 B) $124,513 C) $127,900 D) $131,100

D) $131,100 Explanation: Flexible Budget Wells serviced (q) 31 Expenses: Employee salaries and wages ($41,300 + $1,000q) $72,300 Servicing materials ($600q) 18,600 Other expenses ($40,200) 40,200 Total expenses $131,100

Dreckman Corporation is a service company that measures its output by the number of customers served. The company has provided the following fixed and variable cost estimates that it uses for budgeting purposes and the actual results of operations for August. Fixed Element per Month Variable Element per Customer Served Actual Total for August Revenue $4,000 $125,500 Employee salaries and wages $46,600 $900 $74,700 Travel expenses $600 $18,800 Other expenses $33,100 $33,500 When the company prepared its planning budget at the beginning of August, it assumed that 36 customers would have been served. However, 31 customers were actually served during August. The "Other expenses" in the flexible budget for August would have been closest to: A) $33,500 B) $38,903 C) $28,847 D) $33,100

D) $33,100 Explanation: Flexible Budget Customers served (q) 31 Other expenses ($33,100 fixed) $33,100

All of Gaylord Corporation's sales are on account. Thirty-five percent of the sales on account are collected in the month of sale, 45% in the month following sale, and the remainder are collected in the second month following sale. The following are budgeted sales data for the company: January February March April Total sales $50,000 $60,000 $40,000 $30,000 What is the amount of cash that should be collected in March? A) $24,000 B) $37,000 C) $41,000 D) $51,000

D) $51,000 Explanation: March sales collected in March ($40,000 × 35%) $14,000 February sales collected in March ($60,000 × 45%) $27,000 January sales collected in March ($50,000 × 20%) $10,000 Total cash collections in March $51,000

Runyon Inc. reported the following results from last year's operations: Sales $16,800,000 Variable expenses 12,230,000 Contribution margin 4,570,000 Fixed expenses 3,394,000 Net operating income $1,176,000 The company's average operating assets were $7,000,000. Last year's turnover was closest to: A) 0.42 B) 14.29 C) 0.07 D) 2.40

D) 2.40 Explanation: Turnover = Sales ÷ Average operating assets = $16,800,000 ÷ $7,000,000 = 2.40

Tennill Inc. has a $1,400,000 investment opportunity with the following characteristics: Sales $4,480,000 Contribution margin ratio 40% of sales Fixed expenses $1,657,600 The ROI for this year's investment opportunity considered alone is closest to: A) 8.1% B) 128.0% C) 3.0% D) 9.6%

D) 9.6% Explanation: Margin = Net operating income ÷ Sales= $1,877,760 ÷ $19,560,000 = 9.6%

Which of the following would be classified as a product-level activity? A) Setting up a machine for a batch of a standard product. B) Operating a cafeteria for employees. C) Running the Human Resource department. D) Advertising a product.

D) Advertising a product.

Which of the following may appear on a flexible budget performance report? A) An unfavorable activity variance. B) A favorable revenue variance. C) An unfavorable spending variance. D) All of the above may appear on a flexible budget performance report.

D) All of the above may appear on a flexible budget performance report.

Which of the following would not be included in operating assets in return on investment calculations? A) Cash. B) Accounts Receivable. C) Equipment D) Factory building rented to (and occupied by) another company.

D) Factory building rented to (and occupied by) another company.

Which of the following budgets are prepared before the sales budget? Budgeted Income Statement Direct Labor Budget A) Yes Yes B) Yes No C) No Yes D) No No

D) No No

Which of the following would be classified as a financing activity on the statement of cash flows? A) Paying suppliers for inventory purchases. B) Interest paid to lenders. C) Lending money to another company. D) Repurchasing capital stock from owners.

D) Repurchasing capital stock from owners.

United Industries manufactures a number of products at its highly automated factory. The products are very popular, with demand far exceeding the factory's capacity. To maximize profit, management should rank products based on their: A) gross margin B) contribution margin C) selling price D) contribution margin per unit of the constrained resource

D) contribution margin per unit of the constrained resource

Kinsi Corporation manufactures five different products. All five of these products must pass through a stamping machine in its fabrication department. This machine is Kinsi's constrained resource. Kinsi would make the most profit if it produces the product that: A) uses the least amount of stamping time. B) generates the highest contribution margin per unit. C) generates the highest contribution margin ratio. D) generates the highest contribution margin per stamping machine hour.

D) generates the highest contribution margin per stamping machine hour.

A joint product is: A) any product which consists of several parts. B) any product produced by a company with more than one product line. C) any product involved in a make or buy decision. D) one of several products produced from a common input.

D) one of several products produced from a common input.

Which of the following measures of performance encourages continued expansion by an investment center so long as it is able to earn a return in excess of the minimum required return on average operating assets? A) return on investment B) transfer pricing C) the contribution approach D) residual income

D) residual income

A favorable labor rate variance indicates that A) actual hours exceed standard hours. B) standard hours exceed actual hours. C) the actual rate exceeds the standard rate. D) the standard rate exceeds the actual rate.

D) the standard rate exceeds the actual rate.

When switching from a traditional costing system to an activity-based costing system that contains some batch-level costs: A) the unit product costs of both high and low volume products typically increase. B) the unit product costs of both high and low volume products typically decrease. C) the unit product costs of high volume products typically increase and the unit product costs of low volume products typically decrease. D) the unit product costs of high volume products typically decrease and the unit product costs of low volume products typically increase.

D) the unit product costs of high volume products typically decrease and the unit product costs of low volume products typically increase.

The opportunity cost of making a component part in a factory with excess capacity for which there is no alternative use is: A) the variable manufacturing cost of the component. B) the total manufacturing cost of the component. C) the fixed manufacturing cost of the component. D) zero.

D) zero.

Shoshoni Corporation prepares its statement of cash flows using the indirect method. Which of the following would be added to net income in the operating activities section of the statement? Increase in Accounts Receivable Decrease in Accounts Payable A)YesYes B)YesNo C)NoYes D)NoNo

D)NoNo

Which of the following segment performance measures will decrease if there is an increase in the interest expense for that segment? Return on Investment Residual Income A)YesYes B)NoYes C)YesNo D)NoNo

D)NoNo

Which of the following would be relevant in the decision to sell or throw out obsolete inventory? Direct material cost assigned to the inventory Fixed overhead cost assigned to the inventory A)YesYes B)YesNo C)NoYes D)NoNo

D)NoNo

Amster Corporation has not yet decided on the required rate of return to use in its capital budgeting. This lack of information will prevent Amster from calculating a project's: Payback Net Present Value Internal Rate of Return A)NoNoNo B)YesYesYes C)NoYesYes D)NoYesNo

D)NoYesNo

Down Under Products, Ltd., of Australia has budgeted sales of its popular boomerang for the next four months as follows: Unit SalesApril 50,000 May 75,000 June 90,000 July 80,000 The company is now in the process of preparing a production budget for the second quarter. Past experience has shown that end-of-month inventory levels must equal 10% of the following month's unit sales. The inventory at the end of March was 5,000 units. Required: Prepare a production budget by month and in total, for the second quarter.

Desired units of ending finished goods inventory is 10% of the following month's sales in units.

Which costs will change with a decrease in activity within the relevant range? Total fixed costs and total variable cost.Unit fixed costs and total variable cost.Unit variable cost and unit fixed cost.Unit fixed cost and total fixed cost.

Unit fixed costs and total variable cost.

Three grams of musk oil are required for each bottle of Mink Caress, a very popular perfume made by a small company in western Siberia. The cost of the musk oil is $1.50 per gram. Budgeted production of Mink Caress is given below by quarters for Year 2 and for the first quarter of Year 3: Year 2 Year 3 FirstSecondThirdFourth First Budgeted production, in bottles60,00090,000150,000100,000 70,000 Musk oil has become so popular as a perfume ingredient that it has become necessary to carry large inventories as a precaution against stock-outs. For this reason, the inventory of musk oil at the end of a quarter must be equal to 20% of the following quarter's production needs. Some 36,000 grams of musk oil will be on hand to start the first quarter of Year 2. Required: Prepare a direct materials budget for musk oil, by quarter and in total, for Year 2.

Desired units of ending raw materials inventory: Fourth quarter: 70,000 units × 3 grams per unit × 20% = 42,000 grams.

Northwest Hospital is a full-service hospital that provides everything from major surgery and emergency room care to outpatient clinics. Required: For each of the following costs incurred at Northwest Hospital, indicate whether it would most likely be a direct cost or an indirect cost of the specified cost object.

Ex.Catered food served to patientsA particular patientDirect Cost1.The wages of pediatric nursesThe pediatric departmentDirect Costselected answer correct2.Prescription drugsA particular patientDirect Costselected answer correct3.Heating the hospitalThe pediatric departmentIndirect Costselected answer correct4.The salary of the head of pediatricsThe pediatric departmentDirect Costselected answer correct5.The salary of the head of pediatricsA particular pediatric patientIndirect Costselected answer correct6.Hospital chaplain's salaryA particular patientIndirect Costselected answer correct7.Lab tests by outside contractorA particular patientDirect Costselected answer correct8.Lab tests by outside contractorA particular departmentDirect Cost

Northeast Hospital's Radiology Department is considering replacing an old inefficient X-ray machine with a state-of-the-art digital X-ray machine. The new machine would provide higher quality X-rays in less time and at a lower cost per X-ray. It would also require less power and would use a color laser printer to produce easily readable X-ray images. Instead of investing the funds in the new X-ray machine, the Laboratory Department is lobbying the hospital's management to buy a new DNA analyzer. Required: Classify each item as a differential cost, a sunk cost, or an opportunity cost in the decision to replace the old X-ray machine with a new machine. If none of the categories apply for a particular item, select "None".

Ex.Cost of X-ray film used in the old machineDifferential Cost1.Cost of the old X-ray machineSunk Costselected answer correct2.The salary of the head of the Radiology DepartmentNoneselected answer correct3.The salary of the head of the Laboratory DepartmentNoneselected answer correct4.Cost of the new color laser printerDifferential Costselected answer correct5.Rent on the space occupied by RadiologyNoneselected answer correct6.The cost of maintaining the old machineDifferential Costselected answer correct7.Benefits from a new DNA analyzerOpportunity Costselected answer correct8.Cost of electricity to run the X-ray machinesDifferential Cost Note: The costs of the salaries of the head of the Radiology Department and Laboratory Department and the rent on the space occupied by Radiology are neither differential costs, nor opportunity costs, nor sunk costs. These costs do not differ between the alternatives and therefore are irrelevant in the decision, but they are not sunk costs because they occur in the future.

Silver Company makes a product that is very popular as a Mother's Day gift. Thus, peak sales occur in May of each year, as shown in the company's sales budget for the second quarter given below: AprilMayJuneTotalBudgeted sales (all on account)$300,000$500,000$200,000$1,000,000 From past experience, the company has learned that 20% of a month's sales are collected in the month of sale, another 70% are collected in the month following sale, and the remaining 10% are collected in the second month following sale. Bad debts are negligible and can be ignored. February sales totaled $230,000, and March sales totaled $260,000. Required: 1. Prepare a schedule of expected cash collections from sales, by month and in total, for the second quarter. 2. What is the accounts receivable balance on June 30th?

February sales: $230,000 × 10%$23,000 $23,000March sales: $260,000 × 70%, 10% 182,000$26,000 208,000April sales: $300,000 × 20%, 70%, 10% 60,000 210,000$30,000 300,000May sales: $500,000 × 20%, 70% 100,000 350,000 450,000June sales: $200,000 × 20% 40,000 40,000Total cash collections$265,000$336,000$420,000$1,021,000 Accounts receivable at June 30: From May sales: $500,000 × 10% = $50,000 From June sales: $200,000 × (70% + 10%) = $160,000

Which of the following statements is not correct concerning multiple overhead rate systems? A multiple overhead rate system is more complex than a system based on a single plantwide overhead rate.A multiple overhead rate system is usually more accurate than a system based on a single plantwide overhead rate.A company may choose to create a separate overhead rate for each of its production departments.In departments that are relatively labor-intensive, their overhead costs should be applied to jobs based on machine-hours rather than on direct labor-hours.

In departments that are relatively labor-intensive, their overhead costs should be applied to jobs based on machine-hours rather than on direct labor-hours.

Which of the following is an assumption underlying standard CVP analysis? In multiproduct companies, the sales mix is constant. In manufacturing companies, inventories always change.The price of a product or service is expected to change as volume changes.Fixed expenses will change as volume increases.

In multiproduct companies, the sales mix is constant.

Which of the following statements about using a plantwide overhead rate based on direct labor is correct? Using a plantwide overhead rate based on direct labor-hours will ensure that direct labor costs are correctly traced to jobs.Using a plantwide overhead rate based on direct labor costs will ensure that direct labor costs will be correctly traced to jobs.It is often overly simplistic and incorrect to assume that direct labor-hours is a company's only manufacturing overhead cost driver.The labor theory of value ensures that using a plantwide overhead rate based on direct labor will do a reasonably good job of assigning overhead costs to jobs.

It is often overly simplistic and incorrect to assume that direct labor-hours is a company's only manufacturing overhead cost driver.

Klumper Corporation is a diversified manufacturer of industrial goods. The company's activity-based costing system contains the following six activity cost pools and activity rates: Activity Cost PoolActivity RatesSupporting direct labor$6per direct labor-hourMachine processing$4per machine-hourMachine setups$50per setupProduction orders$90per orderShipments$14per shipmentProduct sustaining$840per product Activity data have been supplied for the following two products: Total Expected Activity K425M67Number of units produced per year2002,000Direct labor-hours80500Machine-hours1001,500Machine setups14Production orders14Shipments110Product sustaining11 Required: How much total overhead cost would be assigned to K425 and M67 using the activity-based costing system?

K425 Supporting direct labor$6per direct labor-hour80direct labor-hours$480Machine processing$4per machine-hour100machine-hours 400Machine setups$50per setup1setup 50Production orders$90per order1order 90Shipments$14per shipment1shipment 14Product sustaining$840per product1product 840Total overhead cost $1,874M67 Supporting direct labor$6per direct labor-hour500direct labor-hours$3,000Machine processing$4per machine-hour1,500machine-hours 6,000Machine setups$50per setup4setups 200Production orders$90per order4orders 360Shipments$14per shipment10shipments 140Product sustaining$840per product1product 840Total overhead cost $10,540

In a job-order costing system, indirect labor cost is usually recorded as a debit to: Manufacturing Overhead.Finished Goods.Work in Process.Cost of Goods Sold.

Manufacturing Overhead.

Luthan Company uses a plantwide predetermined overhead rate of $23.40 per direct labor-hour. This predetermined rate was based on a cost formula that estimated $257,400 of total manufacturing overhead cost for an estimated activity level of 11,000 direct labor-hours. The company incurred actual total manufacturing overhead cost of $249,000 and 10,800 total direct labor-hours during the period. Required: Determine the amount of manufacturing overhead cost that would have been applied to all jobs during the period.

Manufacturing overhead applied $252,720 Actual direct labor-hours (a) 10,800Predetermined overhead rate (b)$23.40Manufacturing overhead applied (a) × (b)$252,720

Assigning manufacturing overhead to a specific job is complicated by all of the below except: Manufacturing overhead is an indirect cost that is either impossible or difficult to trace to a particular job.Manufacturing overhead is incurred only to support some jobs.Manufacturing overhead consists of both variable and fixed costs.The average cost of actual fixed manufacturing overhead expenses will vary depending on how many units are produced in a period.

Manufacturing overhead is incurred only to support some jobs.

If a company increases its selling price by $2 per unit due to an increase in its variable labor cost of $2 per unit, the break-even point in units will: decrease. increase. not change. change but direction cannot be determined.

Not change

Refer to the T-account below: Manufacturing Overhead(2)9,000(12)167,000(3)15,000 (4)80,000 (5)30,000 (6)25,000 159,000 167,000 Bal.8,000 The ending balance of $8,000 represents which of the following? Underapplied overhead.Manufacturing overhead that will be carried over to the next period.Overapplied overhead.A bookkeeping error.

Overapplied overhead.

Harris Fabrics computes its plantwide predetermined overhead rate annually on the basis of direct labor-hours. At the beginning of the year, it estimated that 20,000 direct labor-hours would be required for the period's estimated level of production. The company also estimated $94,000 of fixed manufacturing overhead cost for the coming period and variable manufacturing overhead of $2.00 per direct labor-hour. Harris's actual manufacturing overhead cost for the year was $123,900 and its actual total direct labor was 21,000 hours. Required: Compute the company's plantwide predetermined overhead rate for the year. (Round your answer to 2 decimal places.)

Predetermined overhead rate $6.70 per DLH Y = $94,000 + ($2.00 per DLH)(20,000 DLHs) Estimated fixed manufacturing overhead$94,000Estimated variable manufacturing overhead: $2.00 per DLH × 20,000 DLHs 40,000Estimated total manufacturing overhead cost$134,000 Estimated total manufacturing overhead (a)$134,000 Estimated total direct labor hours (b) 20,000DLHsPredetermined overhead rate (a) ÷ (b)$6.70per DLH

In a job-order costing system that is based on machine-hours, which of the following formulas is correct? Predetermined overhead rate = Actual manufacturing overhead ÷ Actual machine-hoursPredetermined overhead rate = Actual manufacturing overhead ÷ Estimated machine-hoursPredetermined overhead rate = Estimated manufacturing overhead ÷ Estimated machine-hoursPredetermined overhead rate = Estimated manufacturing overhead ÷ Actual machine-hours

Predetermined overhead rate = Estimated manufacturing overhead ÷ Estimated machine-hours

Which of the following is the correct formula to compute the predetermined overhead rate? Predetermined overhead rate = Estimated total units in the allocation base ÷ Estimated total manufacturing overhead costsPredetermined overhead rate = Estimated total manufacturing overhead costs ÷ Estimated total units in the allocation base.Predetermined overhead rate = Actual total manufacturing overhead costs ÷ Estimated total units in the allocation basePredetermined overhead rate = Estimated total manufacturing overhead costs ÷ Actual total units in the allocation base.

Predetermined overhead rate = Estimated total manufacturing overhead costs ÷ Estimated total units in the allocation base

All of the following statements are correct when referring to process costing except: Process costing would be appropriate for a jeweler who makes custom jewelry to order.A process costing system has the same basic purposes as a job-order costing system.Units produced are indistinguishable from each other.Costs are accumulated by department.

Process costing would be appropriate for a jeweler who makes custom jewelry to order.

Refer to the T-account below: Raw MaterialsBal.15,000(9)75,000(5)85,000 Bal.25,000 Entry (5) could represent which of the following? Payments for raw materials.Requisitions of raw materials to be used in production.Purchases of raw materials.Overhead cost applied to Work in Process.

Purchases of raw materials.

Thermal Rising, Inc., makes paragliders for sale through specialty sporting goods stores. The company has a standard paraglider model, but also makes custom-designed paragliders. Management has designed an activity-based costing system with the following activity cost pools and activity rates: Activity Cost PoolActivity RateSupporting direct labor$26per direct labor-hourOrder processing$284per orderCustom design processing$186per custom designCustomer service$379per customer Management would like an analysis of the profitability of a particular customer, Big Sky Outfitters, which has ordered the following products over the last 12 months: StandardModelCustomDesignNumber of gliders 20 3Number of orders 1 3Number of custom designs 0 3Direct labor-hours per glider 26.35 28.00Selling price per glider$1,850$2,400Direct materials cost per glider$564$634 The company's direct labor rate is $19.50 per hour. Required: Using the company's activity-based costing system, compute the customer margin of Big Sky Outfitters. (Round your intermediate calculations and final answer to the nearest whole dollar amount.)

Sales ($1,850 per standard model glider × 20 standard model gliders + $2,400 per custom designed glider × 3 custom designed gliders) $44,200Costs: Direct materials ($564 per standard model glider × 20 standard model gliders + $634 per custom designed glider × 3 custom designed gliders)$13,182 Direct labor ($19.50 per direct labor-hour × 26.35 direct labor-hours per standard model glider × 20 standard model gliders + $19.50 per direct labor-hour × 28 direct labor-hours per custom designed glider × 3 custom designed gliders) 11,915 Supporting direct labor ($26 per direct labor-hour × 26.35 direct labor-hours per standard model glider × 20 standard model gliders + $26 per direct labor-hour × 28 direct labor-hours per custom designed glider × 3 custom designed gliders) 15,886 Order processing ($284 per order × 4 orders) 1,136 Custom design processing ($186 per custom design × 3 custom designs) 558 Customer service ($379 per customer × 1 customer) 379 43,056Customer margin $1,144

Contribution margin is: Sales less cost of goods sold.Sales less variable production, variable selling, and variable administrative expenses.Sales less variable production expense.Sales less all variable and fixed expenses.

Sales less variable production, variable selling, and variable administrative expenses.

The operations vice president of Security Home Bank has been interested in investigating the efficiency of the bank's operations. She has been particularly concerned about the costs of handling routine transactions at the bank and would like to compare these costs at the bank's various branches. If the branches with the most efficient operations can be identified, their methods can be studied and then replicated elsewhere. While the bank maintains meticulous records of wages and other costs, there has been no attempt thus far to show how those costs are related to the various services provided by the bank. The operations vice president has asked your help in conducting an activity-based costing study of bank operations. In particular, she would like to know the cost of opening an account, the cost of processing deposits and withdrawals, and the cost of processing other customer transactions. The Westfield branch of Security Home Bank has submitted the following cost data for last year: Teller wages$160,000Assistant branch manager salary 75,000Branch manager salary 80,000Total$315,000 Virtually all other costs of the branch—rent, depreciation, utilities, and so on—are organization-sustaining costs that cannot be meaningfully assigned to individual customer transactions such as depositing checks. In addition to the cost data above, the employees of the Westfield branch have been interviewed concerning how their time was distributed last year across the activities included in the activity-based costing study. The results of those interviews appear below: Distribution of Resource Consumption Across Activities Opening AccountsProcessing Deposits and WithdrawalsProcessing Other Customer TransactionsOther ActivitiesTotalsTeller wages5%65%20%10%100%Assistant branch manager salary15%5%30%50%100%Branch manager salary5%0%10%85%100% Required: Prepare the first-stage allocation for the activity-based costing study.

Teller wages: Opening accounts = $160,000 × 5% = $8,000 Processing deposits and withdrawals = $160,000 × 65% = $104,000 Processing other customer transactions = $160,000 × 20% = $32,000 Other activities = $160,000 × 10% = $16,000 Assistant branch manager salary: Opening accounts = $75,000 × 15% = $11,250 Processing deposits and withdrawals = $75,000 × 5% = $3,750 Processing other customer transactions = $75,000 × 30% = $22,500 Other activities = $75,000 × 50% = $37,500 Branch manager salary: Opening accounts = $80,000 × 5% = $4,000 Processing deposits and withdrawals = $80,000 × 0% = $0 Processing other customer transactions = $80,000 × 10% = $8,000 Other activities = $80,000 × 85% = $68,000

Which of the following would most likely NOT be included as manufacturing overhead in a furniture factory? The cost of the glue in a chair.The amount paid to the individual who stains a chair.The workman's compensation insurance of the supervisor who oversees production.The factory utilities of the department in which production takes place.

The amount paid to the individual who stains a chair.

Which of the following is true of a company that uses absorption costing? Net operating income fluctuates directly with changes in sales volume.Fixed production and fixed selling costs are considered to be product costs.Unit product costs can change as a result of changes in the number of units manufactured.Variable selling expenses are included in product costs.

Unit product costs can change as a result of changes in the number of units manufactured.

Which of the following is true regarding the contribution margin ratio of a company that produces only a single product? As fixed expenses decrease, the contribution margin ratio increases.The contribution margin ratio multiplied by the selling price per unit equals the contribution margin per unit. The contribution margin ratio will decline as unit sales decline.The contribution margin ratio equals the selling price per unit less the variable expense ratio.

The contribution margin ratio multiplied by the selling price per unit equals the contribution margin per unit.

When using data from a segmented income statement, the dollar sales for a segment to break even is equal to: Multiple ChoiceTraceable fixed expenses ÷ Segment CM ratioCommon fixed expenses ÷ Segment CM ratio(Traceable fixed expenses + Common fixed expenses) ÷ Segment CM ratioNon-traceable fixed expenses ÷ Segment CM ratio

Traceable fixed expenses ÷ Segment CM ratio

Under the weighted-average method, the cost of units transferred out of a department is computed as follows for a cost category: Costs added during the period + Cost of beginning work in process inventoryUnits transferred to the next department × Cost per equivalent unitUnits in ending work in process inventory × Cost per equivalent unitCost of ending work in process inventory − Cost of beginning work in process inventory

Units transferred to the next department × Cost per equivalent unit

In a job-order costing system, manufacturing overhead applied is recorded as a debit to: Raw Materials inventory.Finished Goods inventory.Work in Process inventory.Cost of Goods Sold.

Work in Process inventory.

Which of the following journal entries would be used to record direct labor costs in a company having two processing departments (Department A and Department B)? Work in ProcessXXX Salaries and Wages Payable XXX Salaries and Wages ExpenseXXX Salaries and Wages Payable XXX Work in Process-Department AXXX Work in Process-Department BXXX Salaries and Wages Payable XXX Salaries and Wages PayableXXX Work in Process XXX

Work in Process-Department AXXX Work in Process-Department BXXX Salaries and Wages Payable XXX

Quality Brick Company produces bricks in two processing departments—Molding and Firing. Information relating to the company's operations in March follows: Raw materials used in production: Molding Department, $23,000; and Firing Department, $8,000. Direct labor costs incurred: Molding Department, $12,000; and Firing Department, $7,000. Manufacturing overhead was applied: Molding Department, $25,000; and Firing Department, $37,000. Unfired, molded bricks were transferred from the Molding Department to the Firing Department. According to the company's process costing system, the cost of the unfired, molded bricks was $57,000. Finished bricks were transferred from the Firing Department to the finished goods warehouse. According to the company's process costing system, the cost of the finished bricks was $103,000. Finished bricks were sold to customers. According to the company's process costing system, the cost of the finished bricks sold was $101,000. Required: Prepare journal entries to record items (a) through (f) above. (If no entry is required for a transaction/event, select "No journal entry required" in the first account field.)

a WIPm 23,000 WIPf 8,000 RM 31,000 b WIPm 12,000 WIPf 7,000 Wages Pay 19,000 c WIPm 25,000 WIPf 37,000 Man Over 62,000 d WIPf 57,000 WIPm 57,000 e Finished G 103,000 WIPf 103,000 f COGS 101,000 Finished G 101,000

An example of a committed fixed cost is: management training seminars.a long-term equipment lease.research and development.advertising.

a long-term equipment lease.

If the contribution margin is not sufficient to cover fixed expenses: total profit equals total expenses. contribution margin is negative. a loss occurs. variable expenses equal contribution margin.

a loss occurs.

Mark is an engineer who has designed a telecommunications device. He is convinced that there is a big potential market for the device. Accordingly, he has decided to quit his present job and start a company to manufacture and market the device. The cost of the raw materials that will be used in manufacturing the computer board is: a sunk cost.a fixed cost.a period cost.a variable cost

a variable cost

The salary paid to the president of a company would be classified on the income statement as a(n): administrative expense.direct labor cost.manufacturing overhead cost.selling expense.

administrative expense.

Generally speaking, net operating income under variable and absorption costing will: always be equal.never be equal.be equal only when production and sales are equal.be equal only when production exceeds sales.

be equal only when production and sales are equal.

Weller Company's budgeted unit sales for the upcoming fiscal year are provided below: 1st Quarter2nd Quarter3rd Quarter4th QuarterBudgeted unit sales15,00016,00014,00013,000 The company's variable selling and administrative expense per unit is $2.50. Fixed selling and administrative expenses include advertising expenses of $8,000 per quarter, executive salaries of $35,000 per quarter, and depreciation of $20,000 per quarter. In addition, the company will make insurance payments of $5,000 in the first quarter and $5,000 in the third quarter. Finally, property taxes of $8,000 will be paid in the second quarter. Required: Prepare the company's selling and administrative expense budget for the upcoming fiscal year. (Round "Per Unit" answers to 2 decimal places.)

budgeted unit sales 15,000 16,000 14,000 13,000 58,000 var sell admin unit 2.50 2.50 2.50 2.50 2.50 var sell admin 37,500 40,000 35,000 32,500 145,000 fixed sell admin advertising 8,000 8,000 8,000 8,000 32,000 exec salary 35,000 35,000 35,000 35,000 140,000 insurance 5,000 5,000 10,000 property taxes 8,000 8,000 depreciation 20,000 20,000 20,000 20,000 80,000 total fix sell admin 68,000 71,000 68,000 63,000 270,000 total sell admin 105,500 111,000 103,000 95,500 415,000 less depreciation 20,000 20,000 20,000 20,000 80,000 cash disburse sell admin 85,500 91,000 83,000 75,500 335,000

Direct costs: are incurred to benefit a particular accounting period.are incurred due to a specific decision.can be easily traced to a particular cost object. are the variable costs of producing a product.

can be easily traced to a particular cost object.

Jurvin Enterprises is a manufacturing company that had no beginning inventories. A subset of the transactions that it recorded during a recent month is shown below. $94,000 in raw materials were purchased for cash. $89,000 in raw materials were used in production. Of this amount, $78,000 was for direct materials and the remainder was for indirect materials.. Total labor wages of $132,000 were incurred and paid. Of this amount, $112,000 was for direct labor and the remainder was for indirect labor. Additional manufacturing overhead costs of $143,000 were incurred and paid. Manufacturing overhead of $152,000 was applied to production using the company's predetermined overhead rate. All of the jobs in process at the end of the month were completed. All of the completed jobs were shipped to customers. Any underapplied or overapplied overhead for the period was closed to Cost of Goods Sold. Required: 1. Post the above transactions to T-accounts. 2. Determine the adjusted cost of goods sold for the period.

cash Beg. Bal. 94,000 a. 132,000 c. 143,000 d. End. Bal. 369,000 raw Beg. Bal. 0 a. 94,000 89,000 b. End. Bal. 5,000 Work in Process Beg. Bal. 0 b. 78,000 c. 112,000 e. 152,000 342,000 f. End. Bal. Finished Goods Beg. Bal. 0 f. 342,000 342,000 g. End. Bal. Manufacturing Overhead Beg. Bal. b. 11,000 c. 20,000 d. 143,000 152,000 e. 22,000 h. End. Bal. Cost of Goods Sold Beg. Bal. g. 342,000 h. 22,000 End. Bal. 364,000 2. The adjusted cost of goods sold is shown above as the ending balance in the Cost of Goods Sold T-account ($364,000).

Under a job-order costing system, the dollar amount transferred from Work in Process to Finished Goods is the sum of the costs charged to all jobs: started in process during the period.in process during the period.completed and sold during the period.completed during the period.

completed during the period.

Data concerning a recent period's activity in the Prep Department, the first processing department in a company that uses process costing, appear below: MaterialsConversionEquivalent units in ending work in process inventory 2,000 800 Cost per equivalent unit$13.86 $4.43 A total of 20,100 units were completed and transferred to the next processing department during the period. Required: 1. Compute the cost of ending work in process inventory for materials, conversion, and in total. 2. Compute the cost of the units completed and transferred out for materials, conversion, and in total.

cost of end WIP materials 27,720.00 conversion 3,544.00 total31,264.00 cost of units out materials278,586.00 conversion89,043.00 total 367,629.00 1. MaterialsConversionTotalEnding work in process inventory: Equivalent units 2,000 800 Cost per equivalent unit$13.86 $4.43 Cost of ending work in process inventory$27,720 $3,544 $31,264 2. MaterialsConversionTotalUnits completed and transferred out: Units transferred to the next department 20,100 20,100 Cost per equivalent unit$13.86 $4.43 Cost of units transferred out$278,586 $89,043 $367,629

Superior Micro Products uses the weighted-average method in its process costing system. Data for the Assembly Department for May appear below: MaterialsLaborOverheadWork in process, May 1$18,000$5,500$27,500Cost added during May$238,900$80,300$401,500Equivalent units of production 35,000 33,000 33,000 Required: Compute the cost per equivalent unit for materials, labor, overhead, and in total. (Round your answers to 2 decimal places.)

cost per unit materials:7.34 labor:2.60 overhead:13.00 total:22.94 Cost of beginning work in process inventory$18,000$5,500$27,500Cost added during the period 238,900 80,300 401,500Total cost (a)$256,900$85,800$429,000Equivalent units of production (b) 35,000 33,000 33,000Cost per equivalent unit (a) ÷ (b)$7.34$2.60$13.00 Materials$7.34Labor 2.60Overhead 13.00Total cost per equivalent unit$22.94

When computing the cost per equivalent unit, the weighted-average method of process costing considers: costs incurred during the current period only.costs incurred during the current period plus cost of ending work in process inventory.costs incurred during the current period plus cost of beginning work in process inventory.costs incurred during the current period less cost of beginning work in process inventory.

costs incurred during the current period plus cost of beginning work in process inventory.

Maria Am Corporation uses the weighted-average method in its process costing system. The Baking Department is one of the processing departments in its strudel manufacturing facility. In June in the Baking Department, the cost of beginning work in process inventory was $3,570, the cost of ending work in process inventory was $2,860, and the cost added to production was $43,120. Required: Prepare a cost reconciliation report for the Baking Department for June.

costs to be accounted for cost beg WIP 3,570 cost added to prod during 43,120 total cost to be accounted for 46,690 costs accounted for as follows cost end WIP 2,860 cost units compl and trans 43,830 total cost accounted for 46,690 Cost of beginning work in process inventory+Costs added to production during the period=Cost of ending work in process inventory+Cost of units completed and transferred out $3,570 + $43,120=$2,860+Cost of units completed and transferred out Cost of units completed and transferred out=$3,570 + $43,120 − $2,860 Cost of units completed and transferred out=$43,830

Fixed costs expressed on a per unit basis: increase with increases in activity.decrease with increases in activity.are not affected by activity.should be ignored in making decisions since they cannot change.

decrease with increases in activity.

All of the following are examples of product costs except: depreciation on the company's retail outlets.salary of the plant manager.insurance on the factory equipment.rental costs of factory equipment.

depreciation on the company's retail outlets.

Which of the following costs at a manufacturing company would be treated as a product cost under variable costing? direct material cost property taxes on the factory building sales manager's salary sales commissions

direct material cost

Prime cost consists of: direct labor and manufacturing overhead.direct materials and manufacturing overhead.direct materials and direct labor.direct materials, direct labor and manufacturing overhead.

direct materials and direct labor.

When sales exceed production and the company uses the LIFO inventory flow assumption, the net operating income reported under variable costing generally will be: less than net operating income reported under absorption costing.greater than net operating income reported under absorption costing.equal to net operating income reported under absorption costing.higher or lower because no generalization can be made.

greater than net operating income reported under absorption costing.

A $2.00 increase in a product's variable expense per unit accompanied by a $2.00 increase in its selling price per unit will: decrease the degree of operating leverage. decrease the contribution margin. have no effect on the break-even volume. have no effect on the contribution margin ratio.

have no effect on the break-even volume.

Assume there was no beginning work in process inventory and the ending work in process inventory is 70% complete with respect to conversion costs. Under the weighted-average method, the number of equivalent units of production with respect to conversion costs would be: the same as the units completed.less than the units completed.the same as the units started during the period.less than the units started during the period.

less than the units started during the period.

If sales volume increases and all other factors remain constant, then the: contribution margin ratio will increase.break-even point will decrease.margin of safety will increase. net operating income will decrease.

margin of safety will increase.

Clonex Labs, Inc., uses the weighted-average method in its process costing system. The following data are available for one department for October: Percent Completed UnitsMaterialsConversionWork in process, October 130,00065%30%Work in process, October 3115,00080%40% The department started 175,000 units into production during the month and transferred 190,000 completed units to the next department. Required: Compute the equivalent units of production for October.

materials 202,000 conversion 196,000 Weighted-Average Method: Equivalent Units of Production MaterialsConversionUnits transferred out190,000190,000Equivalent units in ending work in process inventory: 15,000 units × 80%12,000 15,000 units × 40% 6,000Equivalent units of production202,000 196,000

Allocating common fixed expenses to business segments: may cause managers to erroneously discontinue business segments.may cause managers to erroneously keep business segments that should be dropped.ensures that all costs are covered.helps managers make good decisions.

may cause managers to erroneously discontinue business segments.

Dizzy Amusement Park is open from 8:00 am till midnight every day of the year. Dizzy charges its patrons a daily entrance fee of $30 per person which gives them unlimited access to all of the park's 35 rides. For liability insurance, Dizzy pays a set monthly fee plus a small additional amount for every patron entering the park. The cost of liability insurance would best be described as a: fixed cost.mixed cost. step-variable cost.true variable cost

mixed cost

If the degree of operating leverage is 4, then a one percent change in quantity sold should result in a four percent change in: unit contribution margin.revenue.variable expense.net operating income.

net operating income

Which of the following would not affect the break-even point? Multiple Choice number of units sold variable expense per unit total fixed expense selling price per unit

number of units sold

In process costing, a separate work in process account is kept for each: individual order.equivalent unit.processing department.cost category (i.e., materials, conversion cost).

processing department.

When unit sales are constant, but the number of units produced fluctuates and everything else remains the same, net operating income under variable costing will: fluctuate in direct proportion to changes in production.remain constant.fluctuate inversely with changes in production.be greater than net operating income under absorption costing.

remain constant.

Royal Lawncare Company produces and sells two packaged products—Weedban and Greengrow. Revenue and cost information relating to the products follow: Product WeedbanGreengrowSelling price per unit$6.00$7.50Variable expenses per unit$2.40$5.25Traceable fixed expenses per year$45,000$21,000 Common fixed expenses in the company total $33,000 annually. Last year the company produced and sold 15,000 units of Weedban and 28,000 units of Greengrow. Required: Prepare a contribution format income statement segmented by product lines.

sales 300,000 90,000 210,000 variable exp 183,000 36,000 147,000 cont margin 117,000 54,000 63,000 trace fixed 66,000 45,000 21,000 prod line seg margin 51,000 9,000 42,000 common fixed exp 33,000 noi 18,000 Sales: Weedban: 15,000 units × $6.00 per unit = $90,000. Greengrow: 28,000 units × $7.50 per unit = $210,000. Variable expenses: Weedban: 15,000 units × $2.40 per unit = $36,000. Greengrow: 28,000 units × $5.25 per unit = $147,000.

A cost incurred in the past that is not relevant to any current decision is classified as a(n): period cost.opportunity cost.sunk cost.differential cost.

sunk cost.

All of the following can be differential costs except: variable costs.sunk costs.opportunity costs.fixed costs.

sunk costs.

If manufacturing overhead is underapplied, then: actual manufacturing overhead cost is less than estimated manufacturing overhead cost.the amount of manufacturing overhead cost applied to Work in Process is less than the actual manufacturing overhead cost incurred.the predetermined overhead rate is too high.the Manufacturing Overhead account will have a credit balance at the end of the year.

the amount of manufacturing overhead cost applied to Work in Process is less than the actual manufacturing overhead cost incurred.

The impact on net operating income of a small change in sales for a segment is best predicted by using: the contribution margin ratio.the segment margin.the ratio of the segment margin to sales.net sales less segment fixed costs.

the contribution margin ratio.

Higado Confectionery Corporation has a number of store locations throughout North America. In income statements segmented by store, which of the following would be considered a common fixed cost with respect to the stores? store manager salariesstore building depreciation expensethe cost of corporate advertising aired during the Super Bowlcost of goods sold at each store

the cost of corporate advertising aired during the Super Bowl

Hayworth Corporation has just segmented last year's income statement into its ten product lines. The chief executive officer (CEO) is curious as to what effect dropping one of the product lines at the beginning of last year would have had on overall company profit. What is the best number for the CEO to look at to determine the effect of this elimination on the net operating income of the company as a whole? the product line's sales dollarsthe product line's contribution marginthe product line's segment marginthe product line's segment margin minus an allocated portion of common fixed expenses

the product line's segment margin

A reason why absorption costing income statements are sometimes difficult to interpret is that: they omit variable expenses entirely in computing net operating income.they shift portions of fixed manufacturing overhead from period to period according to changing levels of inventories.they include all fixed manufacturing overhead on the income statement each year as a period cost.they ignore inventory levels in determining cost of goods sold.

they shift portions of fixed manufacturing overhead from period to period according to changing levels of inventories.

Which of the following is correct? The break-even point occurs on the CVP graph where: total profit equals total expenses.total profit equals total fixed expenses. total contribution margin equals total fixed expenses. total variable expenses equal total contribution margin.

total contribution margin equals total fixed expenses.

In the standard cost formula Y = a + bX, what does the "Y" represent? total cost. total fixed cost. total variable cost. variable cost per unit.

total cost

Malcolm Company uses a weighted-average process costing system. All materials at Malcolm are added at the beginning of the production process. The equivalent units for materials at Malcolm would be the sum of: units in ending work in process and the units started.units in beginning work in process and the units started.units in ending work in process and the units started and completed.units in beginning work in process and the units started and completed.

units in beginning work in process and the units started.

Jorgansen Lighting, Inc., manufactures heavy-duty street lighting systems for municipalities. The company uses variable costing for internal management reports and absorption costing for external reports to shareholders, creditors, and the government. The company has provided the following data: Year 1Year 2Year 3Inventories: Beginning (units)200170180Ending (units)170180220Variable costing net operating income$1,080,400$1,032,400$996,400 The company's fixed manufacturing overhead per unit was constant at $560 for all three years. Required: 1. Calculate each year's absorption costing net operating income. (Enter any losses or deductions as a negative value.)

var cost noi 1) 1,080,400 2) 1,032,400 3) 996,400 add/ded fix man over 1) (16,800) 2) 5,600 3) 22,400 aborp cost noi 1) 1,063,600 2) 1,038,000 3) 1,018,800 Beginning inventories 200 170 180 Ending inventories 170 180 220 Change in inventories (30) 10 40 Fixed manufacturing overhead in ending inventories (@$560 per unit)$95,200 $100,800 $123,200 Fixed manufacturing overhead in beginning inventories (@$560 per unit) 112,000 95,200 100,800 Fixed manufacturing overhead deferred in (released from) inventories (@$560 per unit)$(16,800)$5,600 $22,400 Variable costing net operating income$1,080,400 $1,032,400 $996,400 Add (deduct) fixed manufacturing overhead cost deferred in (released from) inventory under absorption costing (16,800) 5,600 22,400 Absorption costing net operating income$1,063,600 $1,038,000 $1,018,800

Assuming that direct labor is a variable cost, the primary difference between the absorption and variable costing is that: variable costing treats only direct materials and direct labor as product cost while absorption costing treats direct materials, direct labor, and the variable portion of manufacturing overhead as product costs. variable costing treats direct materials, direct labor, the variable portion of manufacturing overhead, and an allocated portion of fixed manufacturing overhead as product costs while absorption costing treats only direct materials, direct labor, and the variable portion of manufacturing overhead as product costs. variable costing treats only direct materials, direct labor, the variable portion of manufacturing overhead, and the variable portion of selling and administrative expenses as product cost while absorption costing treats direct materials, direct labor, the variable portion of manufacturing overhead, and an allocated portion of fixed manufacturing overhead as product costs. variable costing treats only direct materials, direct labor, and the variable portion of manufacturing overhead as product costs while absorption costing treats direct materials, direct labor, the variable portion of manufacturing overhead, and an allocated portion of fixed manufacturing overhead as product costs.

variable costing treats only direct materials, direct labor, and the variable portion of manufacturing overhead as product costs while absorption costing treats direct materials, direct labor, the variable portion of manufacturing overhead, and an allocated portion of fixed manufacturing overhead as product costs.

The costing method that treats all fixed costs as period costs is: absorption costing.job-order costing.variable costing.process costing.

variable costing.

A cost that would be included in product costs under both absorption costing and variable costing is: supervisory salaries.factory rent.variable manufacturing costs.variable selling expenses.

variable manufacturing costs

Within the relevant range, variable costs can be expected to: vary in total in direct proportion to changes in the activity level.remain constant in total as the activity level changes.increase on a per unit basis as the activity level increases.increase on a per unit basis as the activity level decreases.

vary in total in direct proportion to changes in the activity level.

A process costing system is employed in those situations where: many different products, jobs, or batches of production are being produced each period.where manufacturing involves a single, homogeneous product that flows evenly through the production process on a continuous basis.a service is performed such as in a law firm or an accounting firm.full or absorption cost approach is not employed.

where manufacturing involves a single, homogeneous product that flows evenly through the production process on a continuous basis.


Conjuntos de estudio relacionados

PETER CHEN EXAM 2 (CH 5,6,7,8) - 1

View Set

Chapter 11: Childhood and Neurodevelopmental Disorders

View Set

8.3 Beyond the Mississippi: Louisiana Purchase and the Expedition of Lewis and Clark

View Set

Chapter 1: What is Plant Biology?

View Set

Chapter 12 Quiz, Chapter 13 Quiz, Chapter 14 Quiz, Chapter 15 Quiz, Chapter 16 Quiz, Chapter 17 Quiz, Chapter 18 Quiz, Part III Test

View Set

Principle of Accountant II - Final Exam

View Set